You are on page 1of 230

Cambridge International Examinations

Cambridge International General Certificate of Secondary Education


* 3 4 2 3 1 3 5 5 8 6 *

BIOLOGY 0610/43
Paper 4 Theory (Extended) May/June 2017
1 hour 15 minutes
Candidates answer on the Question Paper.
No Additional Materials are required.

READ THESE INSTRUCTIONS FIRST

Write your Centre number, candidate number and name on all the work you hand in.
Write in dark blue or black pen.
You may use an HB pencil for any diagrams or graphs.
Do not use staples, paper clips, glue or correction fluid.
DO NOT WRITE IN ANY BARCODES.

Answer all questions.

Electronic calculators may be used.


You may lose marks if you do not show your working or if you do not use appropriate units.

At the end of the examination, fasten all your work securely together.
The number of marks is given in brackets [ ] at the end of each question or part question.

The syllabus is approved for use in England, Wales and Northern Ireland as a Cambridge International Level 1/Level 2 Certificate.

This document consists of 18 printed pages and 2 blank pages.

DC (CE/JG) 129192/3
© UCLES 2017 [Turn over
2

1 Fig. 1.1 is a diagram of the human heart. The diagram shows the phase during the heart beat
when the atria contract. The blood vessels that carry blood to and from the heart are labelled
A to F.

C D
B

1 left atrium

right atrium

A F

Fig. 1.1

(a) (i) Draw one arrow on Fig. 1.1 to show the pathway taken by blood from a vein into the
right ventricle. [1]

(ii) Identify the letter of the blood vessel that carries blood at the highest pressure and state
its name.

letter ...............

name of the blood vessel ..................................................................................................


[1]

(b) (i) Suggest what causes the valves at 1 and 2 to close during a heart beat.

...........................................................................................................................................

.......................................................................................................................................[1]

(ii) State the function of valves 1 and 2 in the heart.

.......................................................................................................................................[1]

© UCLES 2017 0610/43/M/J/17


3

(c) Fig. 1.1 shows the phase of the heartbeat when the ventricles are filling with blood.

Using Fig. 1.1, describe and explain how the blood travels from the right ventricle to the
lungs.

...................................................................................................................................................

...................................................................................................................................................

...................................................................................................................................................

...................................................................................................................................................

...................................................................................................................................................

...................................................................................................................................................

...................................................................................................................................................

...................................................................................................................................................

...............................................................................................................................................[4]

(d) State the name of the part of the heart labelled 3 and state its role in the mammalian
circulation.

name .........................................................................................................................................

role ............................................................................................................................................

...................................................................................................................................................
[2]

[Total: 10]

© UCLES 2017 0610/43/M/J/17 [Turn over


4

2 Quinoa, Chenopodium quinoa, is a dicotyledonous plant that produces seeds that resemble those
of cereals, such as rice.

(a) State one feature shown by all dicotyledonous plants.

...................................................................................................................................................

...............................................................................................................................................[1]

(b) During seed development, amino acids are converted into storage proteins and proteases.

Protease molecules become active when the seed absorbs water at the start of germination.

Fig. 2.1 shows the formation of a storage protein and a protease in developing quinoa seeds
and the action of protease on the storage protein during germination.

amino acids in
seeds

protein
synthesis in
seeds

part of a storage
protein molecule part of a
protease
molecule
protease breaks
down storage
protein

supply of amino acids in the


germinating seed

Fig. 2.1

© UCLES 2017 0610/43/M/J/17


5

(i) During seed development in quinoa some genes are ‘switched on’.

Define the term gene.

...........................................................................................................................................

...........................................................................................................................................

.......................................................................................................................................[2]

(ii) Describe the differences in structure between the storage protein and the protease
shown in Fig. 2.1.

...........................................................................................................................................

...........................................................................................................................................

...........................................................................................................................................

...........................................................................................................................................

.......................................................................................................................................[2]

(c) State the roles of mRNA and ribosomes in protein synthesis.

mRNA .......................................................................................................................................

...................................................................................................................................................

ribosome ...................................................................................................................................

...................................................................................................................................................
[2]

© UCLES 2017 0610/43/M/J/17 [Turn over


6

(d) Researchers investigated the effect of pH on the activity of the protease in quinoa seeds.

The results are shown in Fig. 2.2.

0.6

0.5

0.4
enzyme activity
/ arbitrary units 0.3

0.2

0.1

0.0
2.0 2.5 3.0 3.5 4.0 4.5 5.0 5.5 6.0 6.5 7.0
pH

Fig. 2.2

(i) State two factors other than pH that would affect enzyme activity.

1 ........................................................................................................................................

2 ........................................................................................................................................
[2]

(ii) Describe the effect of increasing pH on the activity of the protease in quinoa seeds.

...........................................................................................................................................

...........................................................................................................................................

...........................................................................................................................................

...........................................................................................................................................

...........................................................................................................................................

...........................................................................................................................................

...........................................................................................................................................

...........................................................................................................................................

.......................................................................................................................................[3]

© UCLES 2017 0610/43/M/J/17


7

(iii) Explain the activity of protease at pH 4.0 and at pH 7.0.

...........................................................................................................................................

...........................................................................................................................................

...........................................................................................................................................

...........................................................................................................................................

...........................................................................................................................................

...........................................................................................................................................

...........................................................................................................................................

...........................................................................................................................................

.......................................................................................................................................[4]

[Total: 16]

© UCLES 2017 0610/43/M/J/17 [Turn over


8

3 Red blood cells in humans are produced from stem cells.

Fig. 3.1 shows how a red blood cell is produced and becomes specialised.

nucleus

mitochondria
stem cells

stem cell

production of
haemoglobin

nucleus

mature red
blood cell

Fig. 3.1

© UCLES 2017 0610/43/M/J/17


9

(a) Use the information in Fig. 3.1 to describe how red blood cells are produced and explain how
they are adapted to their function.

...................................................................................................................................................

...................................................................................................................................................

...................................................................................................................................................

...................................................................................................................................................

...................................................................................................................................................

...................................................................................................................................................

...................................................................................................................................................

...................................................................................................................................................

...................................................................................................................................................

...................................................................................................................................................

...................................................................................................................................................

...................................................................................................................................................

...............................................................................................................................................[6]

(b) Red blood cells are suspended in the liquid part of the blood.

State the name of the liquid part of the blood.

...............................................................................................................................................[1]

(c) Stem cells are also found under the outer layer of the skin.

Explain why stem cells are found in the skin.

...................................................................................................................................................

...................................................................................................................................................

...................................................................................................................................................

...............................................................................................................................................[2]

© UCLES 2017 0610/43/M/J/17 [Turn over


10

(d) A type of anaemia is caused by a dietary deficiency.

(i) State the nutrient that is deficient in the diet when this type of anaemia occurs.

.......................................................................................................................................[1]

(ii) State two symptoms of anaemia.

1 ........................................................................................................................................

2 ........................................................................................................................................
[2]

(e) Some people have sickle cell anaemia.

Describe the cause of this type of anaemia.

...................................................................................................................................................

...................................................................................................................................................

...................................................................................................................................................

...................................................................................................................................................

...................................................................................................................................................

...................................................................................................................................................

...................................................................................................................................................

...................................................................................................................................................

...............................................................................................................................................[4]

[Total: 16]

© UCLES 2017 0610/43/M/J/17


11

4 Fig. 4.1 is a photograph of a yellow-shouldered Amazon, Amazona barbadensis, a species of


parrot found along the Venezuelan coast of the Caribbean.

Fig. 4.1

(a) State the vertebrate group that includes A. barbadensis and give two features that are used
to classify animals into this group.

vertebrate group .......................................................................................................................

feature 1 ....................................................................................................................................

feature 2 ...................................................................................................................................
[2]

(b) This species is subdivided into several populations on the mainland and on the islands of
Margarita and Bonaire. Scientists believe that yellow-shouldered Amazons rarely travel
between these places.

Explain what biologists mean when they refer to populations of animals, such as
A. barbadensis.

...................................................................................................................................................

...................................................................................................................................................

...................................................................................................................................................

...................................................................................................................................................

...................................................................................................................................................

...............................................................................................................................................[3]

© UCLES 2017 0610/43/M/J/17 [Turn over


12

(c) The number of yellow-shouldered Amazons on Margarita Island had decreased to 700 parrots
by 1989. The population then increased to 1600 parrots by 2009.

Part of this increase was due to the release of captive-bred parrots on the island. This is one
of the few successful release programmes of parrots. A similar release programme in Arizona
in the 1980s of a different species of parrot was not successful.

Suggest why release programmes for captive-bred parrots were not successful.

...................................................................................................................................................

...................................................................................................................................................

...................................................................................................................................................

...................................................................................................................................................

...............................................................................................................................................[2]

(d) Some captive breeding programmes involve very small numbers of animals.

Outline the disadvantages of such programmes.

...................................................................................................................................................

...................................................................................................................................................

...................................................................................................................................................

...................................................................................................................................................

...................................................................................................................................................

...................................................................................................................................................

...............................................................................................................................................[3]

© UCLES 2017 0610/43/M/J/17


13

(e) Many biologists think that it is better to conserve ecosystems rather than individual species,
such as parrots.

Explain the advantages of conserving ecosystems.

...................................................................................................................................................

...................................................................................................................................................

...................................................................................................................................................

...................................................................................................................................................

...................................................................................................................................................

...................................................................................................................................................

...................................................................................................................................................

...................................................................................................................................................

...............................................................................................................................................[3]

[Total: 13]

© UCLES 2017 0610/43/M/J/17 [Turn over


14

5 An investigation studied the effect of fertilisers on grass yield and species diversity in a grassland
ecosystem.

Some plots within the grassland were treated with fertilisers containing nitrogen, magnesium and
phosphate.

The control plots did not have any added fertiliser.

The average yields were:

• plots with fertiliser 1733 g m–2 year –1


• plots without fertiliser (control plots) 1009 g m–2 year –1

(a) (i) Calculate the difference between the average yields of the two plots within the field as a
percentage of the average yield of the control plots.

Show your working and give your answer to the nearest whole number.

............................................................ %
[2]

(ii) Explain why the average yield increased as a result of adding the fertiliser.

...........................................................................................................................................

...........................................................................................................................................

...........................................................................................................................................

...........................................................................................................................................

...........................................................................................................................................

...........................................................................................................................................

.......................................................................................................................................[3]

(iii) State the name of the process that occurs when fertiliser washes off land into rivers and
causes an algal bloom.

.......................................................................................................................................[1]

© UCLES 2017 0610/43/M/J/17


15

The researchers counted the number of different plant species in each of the plots in the grassland
to determine species diversity. Counting started at the beginning of the growing season and was
repeated every three weeks.

The average number of plant species in each type of plot is shown in Fig. 5.1.

25

plots without fertiliser


(control plots)
20 plots with fertiliser

15
species diversity
/ average number of plant
species m–2
10

0
0 3 6 9 12 15 18 21 24
time / weeks

Fig. 5.1

(b) Describe the effect of fertiliser on the plant species diversity shown in Fig. 5.1.

...................................................................................................................................................

...................................................................................................................................................

...................................................................................................................................................

...................................................................................................................................................

...................................................................................................................................................

...................................................................................................................................................

...............................................................................................................................................[3]

© UCLES 2017 0610/43/M/J/17 [Turn over


16

(c) Suggest why some species of plants survive in grassland and others do not.

...................................................................................................................................................

...................................................................................................................................................

...................................................................................................................................................

...................................................................................................................................................

...................................................................................................................................................

...................................................................................................................................................

...............................................................................................................................................[2]

[Total: 11]

© UCLES 2017 0610/43/M/J/17


17

6 Meningitis is a transmissible disease. One form of the disease is caused by the bacterium
Neisseria meningitidis.

(a) Define the term transmissible disease.

...................................................................................................................................................

...................................................................................................................................................

...................................................................................................................................................

...............................................................................................................................................[2]

(b) One effect of meningitis is a slowing down of nerve impulses.

State what is meant by a nerve impulse.

...................................................................................................................................................

...................................................................................................................................................

...................................................................................................................................................

...............................................................................................................................................[2]

(c) The spread of meningitis can be controlled by using vaccines.

(i) Explain how vaccination provides active immunity.

...........................................................................................................................................

...........................................................................................................................................

...........................................................................................................................................

...........................................................................................................................................

...........................................................................................................................................

...........................................................................................................................................

...........................................................................................................................................

...........................................................................................................................................

.......................................................................................................................................[4]

(ii) If meningitis disappears from a country, explain why the vaccine should continue to be
used in that country.

...........................................................................................................................................

...........................................................................................................................................

...........................................................................................................................................

.......................................................................................................................................[2]
© UCLES 2017 0610/43/M/J/17 [Turn over
18

(d) People who have meningitis are treated with injections of antibodies to give them passive
immunity.

(i) Suggest why the antibodies must be injected rather than taking them by mouth.

...........................................................................................................................................

...........................................................................................................................................

...........................................................................................................................................

...........................................................................................................................................

.......................................................................................................................................[2]

(ii) Explain why passive immunity does not give long-term protection against diseases, such
as meningitis.

...........................................................................................................................................

...........................................................................................................................................

...........................................................................................................................................

...........................................................................................................................................

...........................................................................................................................................

...........................................................................................................................................

.......................................................................................................................................[2]

[Total: 14]

© UCLES 2017 0610/43/M/J/17


Cambridge International Examinations
Cambridge International General Certificate of Secondary Education
* 4 9 9 6 1 4 9 6 2 0 *

BIOLOGY 0610/43
Paper 4 Theory (Extended) May/June 2018
1 hour 15 minutes
Candidates answer on the Question Paper.
No Additional Materials are required.

READ THESE INSTRUCTIONS FIRST

Write your Centre number, candidate number and name on all the work you hand in.
Write in dark blue or black pen.
You may use an HB pencil for any diagrams or graphs.
Do not use staples, paper clips, glue or correction fluid.
DO NOT WRITE IN ANY BARCODES.

Answer all questions.

Electronic calculators may be used.


You may lose marks if you do not show your working or if you do not use appropriate units.

At the end of the examination, fasten all your work securely together.
The number of marks is given in brackets [ ] at the end of each question or part question.

This syllabus is approved for use in England, Wales and Northern Ireland as a Cambridge International Level 1/Level 2 Certificate.

This document consists of 17 printed pages and 3 blank pages.

DC (LEG/SW) 145575/4
© UCLES 2018 [Turn over
2

1 Two functions of the alimentary canal are mechanical digestion and chemical digestion.

(a) Outline where and how mechanical digestion occurs in the alimentary canal.

...................................................................................................................................................

...................................................................................................................................................

...................................................................................................................................................

...................................................................................................................................................

...................................................................................................................................................

...................................................................................................................................................

...................................................................................................................................................

...................................................................................................................................................

...............................................................................................................................................[4]

(b) Enzymes catalyse the reactions of chemical digestion. Table 1.1 gives information about
chemical digestion in three parts of the alimentary canal.

Complete Table 1.1.

Table 1.1

part of the alimentary


enzyme substrate product(s)
canal

mouth starch

stomach peptides

fat fatty acids and glycerol

[3]

© UCLES 2018 0610/43/M/J/18


3

(c) Substances that are absorbed from the alimentary canal may enter cells and become part of
the cells.

(i) State the storage carbohydrate made from glucose in liver cells.

.......................................................................................................................................[1]

(ii) State the type of protein used in the immune system that is produced from amino acids
by lymphocytes.

.......................................................................................................................................[1]

(iii) Fat is produced from fatty acids and glycerol by cells in the fatty tissue beneath the skin.

State one function of this layer of fat.

.......................................................................................................................................[1]

[Total: 10]

© UCLES 2018 0610/43/M/J/18 [Turn over


4

2 Fig. 2.1 shows an Arctic wolf, Canis lupus. These wolves are one of the few mammals adapted to
the extreme cold of the tundra in the Canadian Arctic and in Alaska.

Fig. 2.1

(a) (i) State two features, visible in Fig. 2.1, that identify Arctic wolves as mammals.

1 ........................................................................................................................................

2 ........................................................................................................................................
[2]

(ii) Arctic wolves show many adaptive features to a cold environment.

Explain what is meant by the term adaptive feature.

...........................................................................................................................................

...........................................................................................................................................

...........................................................................................................................................

...........................................................................................................................................

...........................................................................................................................................

.......................................................................................................................................[3]

© UCLES 2018 0610/43/M/J/18


5

(b) The food available to animals in the Arctic tundra is limited. There is a short growing season
for plants and the environmental conditions do not favour high rates of photosynthesis and
growth compared with temperate and tropical ecosystems.

State three conditions that limit plant growth rates.

1 ................................................................................................................................................

2 ................................................................................................................................................

3 ................................................................................................................................................
[3]

(c) Arctic wolves are the top carnivores in the food web in the tundra.

Explain why the number of Arctic wolves is so small in this ecosystem.

...................................................................................................................................................

...................................................................................................................................................

...................................................................................................................................................

...................................................................................................................................................

...................................................................................................................................................

...................................................................................................................................................

...................................................................................................................................................

...................................................................................................................................................

...................................................................................................................................................

...................................................................................................................................................

...................................................................................................................................................

...................................................................................................................................................

...................................................................................................................................................

...............................................................................................................................................[6]

[Total: 14]

© UCLES 2018 0610/43/M/J/18 [Turn over


6

3 Fig. 3.1 is a scanning electron micrograph of a vertical section through part of the leaf of a broad
bean plant, Vicia faba.

air spaces

Fig. 3.1

(a) (i) State the names of the tissues labelled A and B.

A ........................................................................................................................................

B ........................................................................................................................................
[2]

(ii) The cells in regions B and C in Fig. 3.1 have a large surface area.

Explain why this is necessary for the functioning of the leaf cells.

...........................................................................................................................................

...........................................................................................................................................

...........................................................................................................................................

...........................................................................................................................................

...........................................................................................................................................

.......................................................................................................................................[3]

(iii) Explain why there are many interconnecting air spaces within the leaf.

...........................................................................................................................................

...........................................................................................................................................

...........................................................................................................................................

.......................................................................................................................................[2]

© UCLES 2018 0610/43/M/J/18


7

(b) When water is in short supply, plants can wilt as shown in Fig. 3.2.

Fig. 3.2

(i) State two conditions that are likely to increase the chances of wilting.

1 ........................................................................................................................................

2 ........................................................................................................................................
[2]

(ii) Explain what happens to the cells of a leaf to cause wilting.

...........................................................................................................................................

...........................................................................................................................................

...........................................................................................................................................

...........................................................................................................................................

...........................................................................................................................................

...........................................................................................................................................

...........................................................................................................................................

...........................................................................................................................................

...........................................................................................................................................

.......................................................................................................................................[4]

© UCLES 2018 0610/43/M/J/18 [Turn over


8

(iii) Wilting may look harmful, but it is often a strategy for survival.

Suggest the advantages to a plant of wilting.

...........................................................................................................................................

...........................................................................................................................................

...........................................................................................................................................

...........................................................................................................................................

...........................................................................................................................................

.......................................................................................................................................[2]

[Total: 15]

© UCLES 2018 0610/43/M/J/18


10

4 (a) The endocrine system in mammals produces hormones.

Define the term hormone.

...................................................................................................................................................

...................................................................................................................................................

...................................................................................................................................................

...................................................................................................................................................

...............................................................................................................................................[2]

(b) The responses of the human body to danger are coordinated by the nervous and endocrine
systems.

Fig. 4.1 shows the sequence of events that occurs in response to a dangerous situation that
is detected by the eyes.

Key:
nerve impulses
hormone

Fig. 4.1

© UCLES 2018 0610/43/M/J/18


11

(i) State the tissue in the eye that converts light energy into nerve impulses.

.......................................................................................................................................[1]

(ii) State the part of the eye that has the highest concentration of light-sensitive cells and
gives the most detailed image.

.......................................................................................................................................[1]

(iii) State the type of neurone that conducts impulses from the eye to the brain.

.......................................................................................................................................[1]

(iv) State the nerve that contains these neurones that conduct impulses from the eye to the
brain.

.......................................................................................................................................[1]

(v) Identify the organ labelled P.

.......................................................................................................................................[1]

(vi) Identify the gland labelled Q.

.......................................................................................................................................[1]

(c) Complete Table 4.1 to describe the effects of the hormone released when a person is in a
dangerous situation.

Table 4.1

organ effect of the hormone

heart

liver

lungs

eyes

[4]

© UCLES 2018 0610/43/M/J/18 [Turn over


12

(d) Explain the advantages of coordinating the response to a dangerous situation using both the
nervous system and the endocrine system.

...................................................................................................................................................

...................................................................................................................................................

...................................................................................................................................................

...................................................................................................................................................

...................................................................................................................................................

...................................................................................................................................................

...............................................................................................................................................[4]

(e) (i) Plants also make hormones.

State the name of one hormone made by plants.

.......................................................................................................................................[1]

(ii) Some plant hormones are manufactured and applied to crops to alter aspects of plant
growth.

Describe how the synthetic plant hormone 2,4-D is used in agriculture.

...........................................................................................................................................

...........................................................................................................................................

...........................................................................................................................................

...........................................................................................................................................

.......................................................................................................................................[2]

[Total: 19]

© UCLES 2018 0610/43/M/J/18


13

5 (a) State the balanced chemical equation for aerobic respiration.

...............................................................................................................................................[2]

(b) Students investigated the rate of respiration of crickets (a type of insect) using a carbon
dioxide sensor and laptop as shown in Fig. 5.1. The sensor was fitted inside an airtight glass
jar. The apparatus was set up in a room with a constant temperature of 17 °C.

Fig. 5.1

The students found that the concentration of carbon dioxide inside the jar increased by
50 ppm in 120 seconds.

Calculate the rate of carbon dioxide production as ppm per second.

Show your working and express your answer to two significant figures.

............................................ ppm s–1 [1]

© UCLES 2018 0610/43/M/J/18 [Turn over


14

(c) After 10 minutes, the students opened the jar by removing the sensor. They left the jar open
for 5 minutes but made sure that the crickets remained in the jar. They then replaced the
sensor and took more readings for another 10 minutes.

State and explain one reason for opening the jar after 10 minutes.

...................................................................................................................................................

...................................................................................................................................................

...................................................................................................................................................

...................................................................................................................................................

...............................................................................................................................................[2]

(d) During the investigation the temperature inside the jar increased. The temperature outside
the jar remained constant.

Explain why the temperature inside the jar increased.

...................................................................................................................................................

...................................................................................................................................................

...................................................................................................................................................

...................................................................................................................................................

...............................................................................................................................................[2]

© UCLES 2018 0610/43/M/J/18


15

(e) Researchers in Chile also investigated the rate of respiration in crickets.

They investigated the effect of temperature and body mass on the rate of respiration. They
measured the rate of oxygen consumption in crickets with different body masses, at different
temperatures.

The researchers’ results are shown in Fig. 5.2.

30
27 °C
25

20
rate of oxygen 17 °C
consumption 15
/ 10–3 cm3 h–1
10

7 °C
5

0
0 10 20 30 40 50 60 70 80
body mass / mg
Fig. 5.2

State two conclusions that can be made from the data in Fig. 5.2 and support each conclusion
with evidence from the graph.

...................................................................................................................................................

...................................................................................................................................................

...................................................................................................................................................

...................................................................................................................................................

...................................................................................................................................................

...................................................................................................................................................

...................................................................................................................................................

...................................................................................................................................................

...............................................................................................................................................[4]

[Total:11]

© UCLES 2018 0610/43/M/J/18 [Turn over


16

6 (a) Fig. 6.1 is a half-flower drawing of pride of Barbados, Caesalpinia pulcherrima.

D
C

Fig. 6.1

Complete Table 6.1 by stating the letter from Fig. 6.1 that indicates the organ where each
function occurs and the name of the organ.

Table 6.1

function letter from Fig. 6.1 name of the organ

meiosis to produce pollen grains

pollination

development of seeds

protection of flower in the bud


[4]

© UCLES 2018 0610/43/M/J/18


17

(b) Fig. 6.2 is a scanning electron micrograph of some pollen grains from wind-pollinated flowers
and insect-pollinated flowers.

magnification ×220

Fig. 6.2

(i) Write the formula that would be used to calculate the actual diameter of pollen grain H.

[1]

(ii) The actual diameter of pollen grain H is 0.082 mm.

Convert this value to micrometres (μm).

Space for working.

..................................................... μm [1]

(iii) Explain how the pollen grain labelled J is adapted for insect pollination.

...........................................................................................................................................

...........................................................................................................................................

...........................................................................................................................................

...........................................................................................................................................

.......................................................................................................................................[2]

© UCLES 2018 0610/43/M/J/18 [Turn over


18

(c) Pollen grains grow tubes, which contain haploid male gamete nuclei.

(i) One of these male gamete nuclei fuses with the female gamete.

State the part of the flower that contains the female gamete.

.......................................................................................................................................[1]

(ii) Define the term haploid nucleus.

...........................................................................................................................................

...........................................................................................................................................

.......................................................................................................................................[1]

(iii) Explain why it is important for gametes to be haploid.

...........................................................................................................................................

...........................................................................................................................................

.......................................................................................................................................[1]

[Total: 11]

© UCLES 2018 0610/43/M/J/18


Cambridge Assessment International Education
Cambridge International General Certificate of Secondary Education
* 1 1 8 0 0 3 8 1 3 2 *

BIOLOGY 0610/43
Paper 4 Theory (Extended) May/June 2019
1 hour 15 minutes
Candidates answer on the Question Paper.
No Additional Materials are required.

READ THESE INSTRUCTIONS FIRST

Write your centre number, candidate number and name on all the work you hand in.
Write in dark blue or black pen.
You may use an HB pencil for any diagrams or graphs.
Do not use staples, paper clips, glue or correction fluid.
DO NOT WRITE IN ANY BARCODES.

Answer all questions.

Electronic calculators may be used.


You may lose marks if you do not show your working or if you do not use appropriate units.

At the end of the examination, fasten all your work securely together.
The number of marks is given in brackets [ ] at the end of each question or part question.

This syllabus is regulated for use in England, Wales and Northern Ireland as a Cambridge International Level 1/Level 2 Certificate.

This document consists of 16 printed pages and 4 blank pages.

DC (ST/CT) 162742/4
© UCLES 2019 [Turn over
2

1 Bacteria are classified in the Prokaryote kingdom.

(a) State two features of animal cells that are not found in bacteria.

1 ................................................................................................................................................

2 ................................................................................................................................................
[2]

(b) The bacterium Bacillus megaterium was grown in the laboratory fermenter shown in Fig. 1.1.

air lock water


sterile air

magnetic
stirrer

bacteria, source of
nitrogen and glucose

Fig. 1.1

(i) Explain why a source of nitrogen and glucose were added to the fermenter.

nitrogen .............................................................................................................................

...........................................................................................................................................

glucose ..............................................................................................................................

...........................................................................................................................................
[2]

(ii) Suggest why it is important to stir the contents of the fermenter continuously.

...........................................................................................................................................

...........................................................................................................................................

...........................................................................................................................................

...........................................................................................................................................

...........................................................................................................................................

...........................................................................................................................................

..................................................................................................................................... [3]

© UCLES 2019 0610/43/M/J/19


3

(c) Samples were taken from the fermenter at frequent intervals and the number of live bacteria
was determined. The results are shown in Fig. 1.2.

800
C
700

600

500
number of
live bacteria D
400
/ million per
cm3 B
300

200

100
A
0
0 10 20 30 40 50 60

time / hours

Fig. 1.2

Describe and explain what happens to the number of live bacteria shown in the stages
labelled A, B, C and D in Fig. 1.2.

...................................................................................................................................................

...................................................................................................................................................

...................................................................................................................................................

...................................................................................................................................................

...................................................................................................................................................

...................................................................................................................................................

...................................................................................................................................................

...................................................................................................................................................

...................................................................................................................................................

...................................................................................................................................................

...................................................................................................................................................

............................................................................................................................................. [6]

[Total: 13]

© UCLES 2019 0610/43/M/J/19 [Turn over


4

2 (a) State the word equation for photosynthesis.

............................................................................................................................................. [2]

(b) Scientists investigated the effect of light intensity on the rate of photosynthesis in the leaves
of eucalyptus trees at two different concentrations of carbon dioxide, A and B.

The results are shown in Fig. 2.1.

rate of photosynthesis
/ μmol per m2 per s

50

45 B

40

35

30

25
A
20

15

10

0
0 250 500 750 1000 1250 1500 1750 2000
light intensity / arbitrary units

Key:
A carbon dioxide concentration
140 ppm
B carbon dioxide concentration
1000 ppm

Fig. 2.1

© UCLES 2019 0610/43/M/J/19


5

(i) Suggest and explain why the scientists kept the temperature of the leaves at 20 °C while
they recorded results.

...........................................................................................................................................

...........................................................................................................................................

...........................................................................................................................................

...........................................................................................................................................

..................................................................................................................................... [2]

(ii) Calculate the percentage increase in the rate of photosynthesis at a light intensity of
1250 arbitrary units when the carbon dioxide concentration was increased from 140 ppm
to 1000 ppm.

Show your working and give your answer to the nearest whole number.

............................................................ %
[3]

(iii) Describe the effect of increasing light intensity on the rate of photosynthesis when the
concentration of carbon dioxide was 140 ppm.

...........................................................................................................................................

...........................................................................................................................................

...........................................................................................................................................

...........................................................................................................................................

...........................................................................................................................................

...........................................................................................................................................

..................................................................................................................................... [3]

© UCLES 2019 0610/43/M/J/19 [Turn over


6

(iv) Explain the effect of increasing light intensity on the rate of photosynthesis when the
concentration of carbon dioxide was 1000 ppm.

Use the term limiting factor in your answer.

...........................................................................................................................................

...........................................................................................................................................

...........................................................................................................................................

...........................................................................................................................................

...........................................................................................................................................

...........................................................................................................................................

...........................................................................................................................................

...........................................................................................................................................

..................................................................................................................................... [4]

[Total: 14]

© UCLES 2019 0610/43/M/J/19


8

3 Cotton, Gossypium hirsutum, is grown for the fibres that form within the fruits after fertilisation, as
shown in Fig. 3.1.

Fibres from the fruits of cotton plants are used in the textile industry.

fibres

Fig. 3.1

Cotton plants have been genetically engineered to produce a protein that is toxic to the caterpillars
of several insect pests. This gives the cotton plants resistance to the pests.

The cry gene for pest resistance was isolated from the bacterium Bacillus thuringiensis and
inserted into the cells of cotton plants as shown in Fig. 3.2.

cry gene

A A
bacterial
DNA plasmid

cotton plant cell


not to scale
Fig. 3.2

© UCLES 2019 0610/43/M/J/19


9

(a) An enzyme cuts the cry gene from the DNA of B. thuringiensis.

(i) State the name of the enzyme that cuts DNA.

..................................................................................................................................... [1]

(ii) State the name of the regions labelled A on Fig. 3.2.

..................................................................................................................................... [1]

(iii) Explain how the DNA is inserted into the plasmid.

...........................................................................................................................................

...........................................................................................................................................

...........................................................................................................................................

...........................................................................................................................................

...........................................................................................................................................

..................................................................................................................................... [3]

(b) The plasmids containing the cry gene are inserted into the cells of cotton plants.

Outline how the cells of cotton plants use the cry gene to make the toxic protein.

...................................................................................................................................................

...................................................................................................................................................

...................................................................................................................................................

...................................................................................................................................................

...................................................................................................................................................

...................................................................................................................................................

...................................................................................................................................................

............................................................................................................................................. [4]

© UCLES 2019 0610/43/M/J/19 [Turn over


10

(c) Outline the advantages to farmers of growing genetically engineered cotton plants that
contain the toxic protein.

...................................................................................................................................................

...................................................................................................................................................

...................................................................................................................................................

...................................................................................................................................................

...................................................................................................................................................

...................................................................................................................................................

...................................................................................................................................................

...................................................................................................................................................

...................................................................................................................................................

...................................................................................................................................................

............................................................................................................................................. [3]

[Total: 12]

© UCLES 2019 0610/43/M/J/19


11

4 Mammals have a double circulation.

(a) State what is meant by the term double circulation.

...................................................................................................................................................

...................................................................................................................................................

............................................................................................................................................. [1]

(b) Table 4.1 shows some information about the functions of the components of blood.

Complete Table 4.1.

Table 4.1

function type of cell

production of antibodies

phagocyte

promotes blood clotting

transports oxygen

[4]

(c) Blood is transported in arteries and veins.

Complete the drawings of the cross-sections of an artery and a vein to show the differences
between these two types of blood vessel. Label the lumen in each drawing.

artery vein

[2]

© UCLES 2019 0610/43/M/J/19 [Turn over


12

(d) A diagram of a mammalian heart and associated blood vessels is shown in Fig. 4.1.

A
L

J D

Fig. 4.1

(i) Sketch arrows on Fig. 4.1 to show the pathway taken by deoxygenated blood from the
heart towards the lungs. [2]

© UCLES 2019 0610/43/M/J/19


13

(ii) Table 4.2 contains statements about the structures visible in Fig. 4.1.

Complete Table 4.2 by:

• stating the name of each structure


• identifying the structure with the corresponding letter from Fig. 4.1.

Table 4.2

statement name of structure letter from Fig. 4.1


chamber that creates the highest
blood pressure
blood vessel containing blood with
the highest concentration of oxygen
structure that prevents blood going
from ventricle to atrium
structure that prevents backflow of
blood from artery to ventricle
chamber that receives blood from
vena cava
[5]

(e) Mammals also have a lymphatic system.

Outline the functions of the lymphatic system.

...................................................................................................................................................

...................................................................................................................................................

...................................................................................................................................................

...................................................................................................................................................

...................................................................................................................................................

...................................................................................................................................................

............................................................................................................................................. [3]

[Total: 17]

© UCLES 2019 0610/43/M/J/19 [Turn over


14

5 Sickle-cell anaemia is an inherited disease.

Fig. 5.1 is a photomicrograph of some blood cells from a person who has sickle-cell anaemia.

a sickle-shaped red blood cell

Fig. 5.1

(a) Explain how red blood cells become sickle-shaped.

...................................................................................................................................................

...................................................................................................................................................

...................................................................................................................................................

...................................................................................................................................................

...................................................................................................................................................

...................................................................................................................................................

............................................................................................................................................. [3]

© UCLES 2019 0610/43/M/J/19


15

(b) Some people who have sickle-cell anaemia have parents who do not have sickle-cell
anaemia.

Explain how people with sickle-cell anaemia inherit the disease.

...................................................................................................................................................

...................................................................................................................................................

...................................................................................................................................................

...................................................................................................................................................

...................................................................................................................................................

...................................................................................................................................................

...................................................................................................................................................

...................................................................................................................................................

...................................................................................................................................................

............................................................................................................................................. [4]

(c) Sickle-cell anaemia is most common in areas of the world where the infectious disease
malaria is found.

Some species of the genus Plasmodium cause malaria in humans.

(i) Define the term species.

...........................................................................................................................................

...........................................................................................................................................

...........................................................................................................................................

...........................................................................................................................................

..................................................................................................................................... [2]

© UCLES 2019 0610/43/M/J/19 [Turn over


16

(ii) The distribution of sickle-cell anaemia is the result of natural selection.

Explain the distribution of the sickle-cell allele in human populations.

...........................................................................................................................................

...........................................................................................................................................

...........................................................................................................................................

...........................................................................................................................................

...........................................................................................................................................

...........................................................................................................................................

...........................................................................................................................................

...........................................................................................................................................

...........................................................................................................................................

..................................................................................................................................... [5]

[Total: 14]

6 Fig. 6.1 is a photomicrograph of part of a cell from the pancreas that produces enzymes that are
released into the small intestine.

×20 000

Fig. 6.1

© UCLES 2019 0610/43/M/J/19


17

(a) (i) Structure Q is part of the nucleus of the cell.

State one function of a nucleus.

...........................................................................................................................................

...........................................................................................................................................

..................................................................................................................................... [1]

(ii) State the names of the structures labelled P and R in Fig. 6.1.

P ........................................................................................................................................

R ........................................................................................................................................
[2]

(b) The structure labelled S transports enzymes to the cell membrane for release into the
pancreatic duct. These structures contain molecules of amylase, trypsin and lipase.

Complete the sentences with the most appropriate words.

Enzymes are made of protein and act as .............................................. because they increase

the rate of chemical reactions, but are not changed in those reactions. Amylase speeds up the

digestion of .............................................. to .............................................. Trypsin continues

the chemical digestion of protein begun by the enzyme .............................................. in the

stomach.

The optimum pH for pancreatic enzymes is greater than pH 7. Bile is produced

by the .............................................. and enters the small intestine, where it

.............................................. stomach acid to provide the appropriate pH. Bile also breaks

down fat by .............................................. to increase the surface area for the action of lipase.

[7]

[Total: 10]

© UCLES 2019 0610/43/M/J/19


Cambridge IGCSE™
* 1 7 9 1 8 2 5 5 2 6 *

BIOLOGY 0610/43
Paper 4 Theory (Extended) May/June 2020

1 hour 15 minutes

You must answer on the question paper.

No additional materials are needed.

INSTRUCTIONS
● Answer all questions.
● Use a black or dark blue pen. You may use an HB pencil for any diagrams or graphs.
● Write your name, centre number and candidate number in the boxes at the top of the page.
● Write your answer to each question in the space provided.
● Do not use an erasable pen or correction fluid.
● Do not write on any bar codes.
● You may use a calculator.
● You should show all your working and use appropriate units.

INFORMATION
● The total mark for this paper is 80.
● The number of marks for each question or part question is shown in brackets [ ].

This document has 20 pages. Blank pages are indicated.

DC (ST/CT) 180793/3
© UCLES 2020 [Turn over
2

1 (a) State three uses of energy in the human body.

1 ................................................................................................................................................

2 ................................................................................................................................................

3 ................................................................................................................................................
[3]

(b) Fig. 1.1 shows part of the digestive system of a human.

diaphragm

B
L

K
C
H

G
D

Fig. 1.1

© UCLES 2020 0610/43/M/J/20


3

Complete Table 1.1. One row has been done for you.

Table 1.1

function name of structure letter from Fig. 1.1

pushes food to the stomach oesophagus A

assimilation of amino acids to produce


plasma proteins

storage of bile

secretion of insulin

absorption of fatty acids and glycerol

secretion of pepsin

digestion of starch

[6]

(c) Describe the role of the liver in the recovery from oxygen debt after strenuous exercise.

...................................................................................................................................................

...................................................................................................................................................

...................................................................................................................................................

...................................................................................................................................................

............................................................................................................................................. [2]

(d) Alcohol is a drug.

Define the term drug.

...................................................................................................................................................

...................................................................................................................................................

............................................................................................................................................. [2]

© UCLES 2020 0610/43/M/J/20 [Turn over


4

(e) (i) State two immediate effects of excessive alcohol on the body.

1 ........................................................................................................................................

2 ........................................................................................................................................
[2]

(ii) State two long-term effects of excessive alcohol on the body.

1 ........................................................................................................................................

2 ........................................................................................................................................
[2]

(f) Pregnant women are advised not to drink alcohol as it may have harmful effects on the fetus.

(i) Outline these harmful effects.

...........................................................................................................................................

...........................................................................................................................................

...........................................................................................................................................

...........................................................................................................................................

..................................................................................................................................... [2]

(ii) State two harmful substances other than alcohol that can cross the placenta.

1 ........................................................................................................................................

2 ........................................................................................................................................
[2]

[Total: 21]

© UCLES 2020 0610/43/M/J/20


5

2 (a) Fig. 2.1 shows the human population of a country between 1910 and 2020.

140

120

100
number of
people 80
/ million
60

40

20

0
1900 1920 1940 1960 1980 2000 2020
year

Fig. 2.1

(i) Calculate the percentage increase in the population of the country between 1940 and
2020.

Space for working.

.............................................................%
[3]

(ii) Describe the factors that could cause the change in the population size between 1940
and 2020, shown in Fig. 2.1.

...........................................................................................................................................

...........................................................................................................................................

...........................................................................................................................................

...........................................................................................................................................

...........................................................................................................................................

...........................................................................................................................................

..................................................................................................................................... [3]

© UCLES 2020 0610/43/M/J/20 [Turn over


6

(b) Some countries have invested in biofuels such as ethanol, biomass and biodiesel.

(i) Describe how ethanol can be made by microorganisms.

...........................................................................................................................................

...........................................................................................................................................

...........................................................................................................................................

...........................................................................................................................................

..................................................................................................................................... [2]

(ii) Some countries use large areas of land to grow maize plants. This crop plant can be
used to produce biofuels.

Discuss the negative impact on the environment of growing large-scale monocultures


of crop plants such as maize.

...........................................................................................................................................

...........................................................................................................................................

...........................................................................................................................................

...........................................................................................................................................

...........................................................................................................................................

...........................................................................................................................................

...........................................................................................................................................

...........................................................................................................................................

..................................................................................................................................... [4]

[Total: 12]

© UCLES 2020 0610/43/M/J/20


8

3 The American writer Ernest Hemingway lived on the island of Key West in Florida, USA in the
1930s. During this time he was given a male cat by a sea captain.

The cat had more toes than usual. This inherited condition is called polydactyly. The allele for
polydactyly is dominant.

(a) Define the term inheritance.

...................................................................................................................................................

...................................................................................................................................................

............................................................................................................................................. [1]

(b) Fig. 3.1 is part of a pedigree diagram for Hemingway’s cats.

1 2

3 4 5 6 7 8 9 10

11 12 13 14 15 16

Key:
17
female cat with normal female cat with
number of toes polydactyly

male cat with normal male cat with


number of toes polydactyly

Fig. 3.1

© UCLES 2020 0610/43/M/J/20


9

(i) State the genotypes of cats 5, 6 and 14 in the pedigree diagram in Fig. 3.1.

Use the letters T and t.

cat 5 ..................................................................................................................................

cat 6 ..................................................................................................................................

cat 14 ................................................................................................................................
[3]

(ii) Explain why none of the offspring of cats 3 and 4 have inherited polydactyly.

Use the information in Fig. 3.1 in your answer.

...........................................................................................................................................

...........................................................................................................................................

..................................................................................................................................... [1]

© UCLES 2020 0610/43/M/J/20 [Turn over


10

(c) Scientists published the results of an investigation into the DNA of cats with and without
polydactyly. They compared the base sequence from a particular region of DNA that controls
the development of the limbs.

Table 3.1 shows the base sequences.

Table 3.1

cats without polydactyly AGA CAC AGA AAT GAG


Hemingway’s cats with polydactyly AGA CAC GGA AAT GAG
cats with polydactyly from Oregon and Missouri in the USA AGA CAC GGA AAT GAG
cats with polydactyly from the UK AGA CAC AGT AAT GAG

(i) Describe how the base sequences of the cats with polydactyly differ from the base
sequence of cats without polydactyly.

...........................................................................................................................................

...........................................................................................................................................

...........................................................................................................................................

...........................................................................................................................................

..................................................................................................................................... [2]

(ii) State the name of the process by which base sequences in DNA are changed.

..................................................................................................................................... [1]

(iii) The base sequences in Table 3.1 provide evidence that indicates which country the male
cat given to Hemingway in the 1930s came from.

Suggest which country this cat came from and give a reason for your choice.

...........................................................................................................................................

...........................................................................................................................................

...........................................................................................................................................

...........................................................................................................................................

..................................................................................................................................... [2]

© UCLES 2020 0610/43/M/J/20


11

(d) Fig. 3.2 shows part of a DNA molecule from a chromosome of a cat.

Complete Fig. 3.2 by writing the letters for the base sequence of the other strand of the DNA
molecule.

T ..........

A ..........

A ..........

T ..........

G ..........

C ..........

G ..........

T ..........

G ..........

Fig. 3.2
[1]

(e) Explain why polydactyly is an example of discontinuous variation.

...................................................................................................................................................

...................................................................................................................................................

...................................................................................................................................................

...................................................................................................................................................

............................................................................................................................................. [2]

[Total: 13]

© UCLES 2020 0610/43/M/J/20 [Turn over


12

4 Xerophytes grow in habitats with low rainfall and soils that often have high concentrations of salts.

Fig. 4.1 shows the xerophyte Yucca treculeana growing on salt flats.

Y. treculeana

Fig. 4.1

(a) (i) Explain how xerophytes, such as Y. treculeana, are adapted to absorb sufficient water in
the conditions in which they live.

...........................................................................................................................................

...........................................................................................................................................

...........................................................................................................................................

...........................................................................................................................................

...........................................................................................................................................

...........................................................................................................................................

..................................................................................................................................... [4]

© UCLES 2020 0610/43/M/J/20


13

(ii) Explain how xerophytes are adapted to reduce water loss to the atmosphere.

...........................................................................................................................................

...........................................................................................................................................

...........................................................................................................................................

...........................................................................................................................................

...........................................................................................................................................

...........................................................................................................................................

..................................................................................................................................... [3]

(iii) Xerophytes often have many defence mechanisms that reduce or prevent herbivores
eating them.

Suggest how xerophytes protect themselves against herbivores.

...........................................................................................................................................

...........................................................................................................................................

...........................................................................................................................................

...........................................................................................................................................

..................................................................................................................................... [2]

(b) Forest ecosystems can be affected by acid rain.

Describe how the production of acid rain and its effects on forest ecosystems can be reduced.

...................................................................................................................................................

...................................................................................................................................................

...................................................................................................................................................

...................................................................................................................................................

...................................................................................................................................................

...................................................................................................................................................

...................................................................................................................................................

...................................................................................................................................................

............................................................................................................................................. [4]

[Total: 13]

© UCLES 2020 0610/43/M/J/20 [Turn over


15

5 Bacteria are classified in the Prokaryote kingdom.

(a) State two features of animal and plant cells that are not found in prokaryotes.

1 ................................................................................................................................................

2 ................................................................................................................................................
[2]

© UCLES 2020 0610/43/M/J/20 [Turn over


16

(b) Methicillin-resistant Staphylococcus aureus (MRSA) is a type of bacterium that is resistant to


some antibiotics.

Fig. 5.1 shows how a population of bacteria may develop antibiotic resistance and how the
antibiotic resistance can be passed from one strain of bacterium to another.

population of pathogenic bacteria A in


patient 1

treatment with antibiotic Z started

population of pathogenic bacteria A in


patient 1 in the middle of treatment
with antibiotic Z

population of pathogenic bacteria A in


patient 1 when treatment with
antibiotic Z was stopped

transmission of bacteria A to patient 2

bacteria A and B in patient 2

not to scale

Key:
bacterium of strain A bacterium of strain B
resistant to antibiotic Z resistant to antibiotic Z
bacterium of strain A bacterium of strain B
not resistant to antibiotic Z not resistant to antibiotic Z
dying bacterium of strain A plasmid

Fig. 5.1

© UCLES 2020 0610/43/M/J/20


17

Explain how resistance to an antibiotic develops in a population of bacteria and spreads in


the human population.

...................................................................................................................................................

...................................................................................................................................................

...................................................................................................................................................

...................................................................................................................................................

...................................................................................................................................................

...................................................................................................................................................

...................................................................................................................................................

...................................................................................................................................................

...................................................................................................................................................

...................................................................................................................................................

...................................................................................................................................................

...................................................................................................................................................

...................................................................................................................................................

...................................................................................................................................................

............................................................................................................................................. [6]

(c) Explain how the development of resistant bacteria, such as MRSA, can be minimised.

...................................................................................................................................................

...................................................................................................................................................

...................................................................................................................................................

...................................................................................................................................................

...................................................................................................................................................

...................................................................................................................................................

...................................................................................................................................................

............................................................................................................................................. [3]

[Total: 11]

© UCLES 2020 0610/43/M/J/20 [Turn over


18

6 In many parts of the world dairy cattle are kept in large barns and reared intensively, as shown in
Fig. 6.1.

Fig. 6.1

(a) Food for cattle that are reared intensively includes cereals, such as maize and barley.

Ecologists have calculated that it is more energy efficient to grow crops for human consumption
than for food for livestock.

Explain why intensive rearing of livestock is not an efficient use of crops.

...................................................................................................................................................

...................................................................................................................................................

...................................................................................................................................................

...................................................................................................................................................

...................................................................................................................................................

...................................................................................................................................................

...................................................................................................................................................

............................................................................................................................................. [3]

© UCLES 2020 0610/43/M/J/20


19

(b) The urine and faeces from cattle kept in barns is removed and treated in the same way as
human sewage to avoid polluting the aquatic environment.

Outline the effects of untreated waste from cattle on the aquatic environment.

...................................................................................................................................................

...................................................................................................................................................

...................................................................................................................................................

...................................................................................................................................................

...................................................................................................................................................

...................................................................................................................................................

...................................................................................................................................................

............................................................................................................................................. [4]

(c) Intensive livestock production could be one way of preventing famine.

Describe the causes of famine.

...................................................................................................................................................

...................................................................................................................................................

...................................................................................................................................................

...................................................................................................................................................

...................................................................................................................................................

...................................................................................................................................................

............................................................................................................................................. [3]

[Total: 10]

© UCLES 2020 0610/43/M/J/20


Cambridge IGCSE™
* 3 6 0 5 4 1 7 5 1 9 *

BIOLOGY 0610/43
Paper 4 Theory (Extended) May/June 2021

1 hour 15 minutes

You must answer on the question paper.

No additional materials are needed.

INSTRUCTIONS
● Answer all questions.
● Use a black or dark blue pen. You may use an HB pencil for any diagrams or graphs.
● Write your name, centre number and candidate number in the boxes at the top of the page.
● Write your answer to each question in the space provided.
● Do not use an erasable pen or correction fluid.
● Do not write on any bar codes.
● You may use a calculator.
● You should show all your working and use appropriate units.

INFORMATION
● The total mark for this paper is 80.
● The number of marks for each question or part question is shown in brackets [ ].

This document has 20 pages. Any blank pages are indicated.

DC (CE/SG) 203218/4
© UCLES 2021 [Turn over
2

1 (a) (i) The main component of milk is a liquid that is an important solvent in the body.

State the name of this solvent.

..................................................................................................................................... [1]

(ii) Table 1.1 shows some of the nutrients found in human milk, the elements that make up
the nutrients, the enzymes that digest them and the products of digestion.

Complete Table 1.1.

Table 1.1

nutrient elements enzyme products of digestion

protein

fat

lactose (milk galactose and glucose


C, H, O
sugar) (simple sugars)

[4]

(b) Milk also contains vitamins and mineral salts.

(i) Explain why vitamin D is important for the nutrition of children.

...........................................................................................................................................

...........................................................................................................................................

...........................................................................................................................................

...........................................................................................................................................

...........................................................................................................................................

...........................................................................................................................................

..................................................................................................................................... [3]

© UCLES 2021 0610/43/M/J/21


3

(ii) Explain why iron is important in the diet.

...........................................................................................................................................

...........................................................................................................................................

...........................................................................................................................................

...........................................................................................................................................

...........................................................................................................................................

..................................................................................................................................... [2]

(c) Rubella and kwashiorkor are two diseases that affect children.

Rubella is a transmissible disease and kwashiorkor is a non-transmissible disease.

(i) Define the term transmissible disease.

...........................................................................................................................................

...........................................................................................................................................

...........................................................................................................................................

..................................................................................................................................... [2]

(ii) Explain why women should be vaccinated against rubella before they reach reproductive
age.

...........................................................................................................................................

...........................................................................................................................................

...........................................................................................................................................

...........................................................................................................................................

...........................................................................................................................................

...........................................................................................................................................

...........................................................................................................................................

..................................................................................................................................... [4]

© UCLES 2021 0610/43/M/J/21 [Turn over


4

(iii) Outline the causes of kwashiorkor.

...........................................................................................................................................

...........................................................................................................................................

...........................................................................................................................................

...........................................................................................................................................

..................................................................................................................................... [2]

[Total: 18]

2 Insulin is a protein made by cells in the pancreas.

Fig. 2.1 shows how insulin is made in a pancreatic cell.

DNA
amino acids

nucleus

amino acids assembled


to form insulin
A

B not to scale

Fig. 2.1

(a) (i) State the name of molecule A in Fig. 2.1.

..................................................................................................................................... [1]

(ii) State the name of structure B in Fig. 2.1.

..................................................................................................................................... [1]

© UCLES 2021 0610/43/M/J/21


5

(iii) Explain the role of molecule A in protein synthesis, as shown in Fig. 2.1.

...........................................................................................................................................

...........................................................................................................................................

...........................................................................................................................................

...........................................................................................................................................

...........................................................................................................................................

...........................................................................................................................................

..................................................................................................................................... [3]

(b) Insulin is secreted from cells in the pancreas when the concentration of glucose in the blood
increases.

Outline the role of insulin.

...................................................................................................................................................

...................................................................................................................................................

...................................................................................................................................................

...................................................................................................................................................

...................................................................................................................................................

...................................................................................................................................................

...................................................................................................................................................

............................................................................................................................................. [4]

(c) State the names of two substances, other than insulin, that are secreted by the pancreas.

1 ................................................................................................................................................

2 ................................................................................................................................................
[2]

[Total: 11]

© UCLES 2021 0610/43/M/J/21 [Turn over


6

3 Scientists measured the rate of photosynthesis in the leaves of a sunflower plant, Helianthus
annuus.

The scientists used the apparatus shown in Fig. 3.1 to measure the rate of photosynthesis.

direction of air flow

air flow regulator chamber 1 chamber 2 chamber 3

temperature 20 °C
CO2 360 ppm

CO2 400 ppm timer


00:00

Fig. 3.1

(a) The apparatus in Fig. 3.1 maintains a constant temperature and a constant humidity.

(i) Explain why temperature has an effect on the rate of photosynthesis.

...........................................................................................................................................

...........................................................................................................................................

...........................................................................................................................................

...........................................................................................................................................

...........................................................................................................................................

...........................................................................................................................................

...........................................................................................................................................

...........................................................................................................................................

..................................................................................................................................... [4]

© UCLES 2021 0610/43/M/J/21


7

(ii) Explain why the rate of photosynthesis will decrease if the humidity in the apparatus
becomes very low.

...........................................................................................................................................

...........................................................................................................................................

...........................................................................................................................................

...........................................................................................................................................

...........................................................................................................................................

...........................................................................................................................................

..................................................................................................................................... [3]

(iii) The apparatus was left for 15 minutes.

Explain how the scientists would use the readings for the concentration of carbon dioxide
in chambers 1 and 3 to calculate the rate of photosynthesis.

...........................................................................................................................................

...........................................................................................................................................

...........................................................................................................................................

...........................................................................................................................................

..................................................................................................................................... [2]

© UCLES 2021 0610/43/M/J/21 [Turn over


8

(b) The scientists carried out another investigation using the same apparatus at different
temperatures.

They measured the rate of uptake of carbon dioxide in the light and then they measured the
rate of release of carbon dioxide in the dark.

The results are shown in Fig. 3.2.

160

140
CO2 uptake
120 in the light

rate of carbon 100


dioxide uptake
and release 80
/ μg per hour
per cm2 of leaf
60

CO2 release
40
in the dark

20

0
15 20 25 30 35 40
temperature / °C

Fig. 3.2

(i) Use the information in Fig. 3.2 to complete Table 3.1.

Table 3.1

temperature rate of uptake of carbon dioxide in rate of release of carbon dioxide in


/ °C the light / μg per hour per cm2 of leaf the dark / μg per hour per cm2 of leaf

20

35 90 45

[2]

© UCLES 2021 0610/43/M/J/21


9

(ii) The scientists determined that photosynthesis in the leaves at 35 °C used carbon dioxide
at a rate of 135 μg per hour per cm2 of leaf. Fig. 3.2 shows that the rate of carbon dioxide
uptake at 35 °C was 90 μg per hour per cm2 of leaf.

Explain why the rate at which carbon dioxide is used in photosynthesis is greater than
the rate of carbon dioxide uptake.

...........................................................................................................................................

...........................................................................................................................................

...........................................................................................................................................

...........................................................................................................................................

..................................................................................................................................... [2]

(iii) Explain why the results in Table 3.1 are expressed as ‘per cm2 of leaf’ rather than ‘per
leaf’.

...........................................................................................................................................

...........................................................................................................................................

..................................................................................................................................... [1]

© UCLES 2021 0610/43/M/J/21 [Turn over


11

(c) Some crops are grown in controlled environments. Glasshouses are an example of a
controlled environment. Fig. 3.3 shows a glasshouse where lettuces are growing.

Fig. 3.3

Discuss the advantages of growing crops in glasshouses.

...................................................................................................................................................

...................................................................................................................................................

...................................................................................................................................................

...................................................................................................................................................

...................................................................................................................................................

...................................................................................................................................................

...................................................................................................................................................

...................................................................................................................................................

...................................................................................................................................................

...................................................................................................................................................

...................................................................................................................................................

...................................................................................................................................................

...................................................................................................................................................

...................................................................................................................................................

...................................................................................................................................................

............................................................................................................................................. [6]

[Total: 20]

© UCLES 2021 0610/43/M/J/21 [Turn over


12

4 There are three species of rhinoceros in Asia. Fig. 4.1 shows an Indian rhinoceros, Rhinoceros
unicornis, in a national park being observed by tourists who are riding on an elephant.

Fig. 4.1

The International Union for Conservation of Nature (IUCN) collects data on animal species and
places them into categories based on estimated numbers.

Table 4.1 summarises the conservation status of the three species of rhinoceros, as available in
2020.

Table 4.1

approximate approximate
species location number in number in IUCN category
the wild captivity
Indian rhinoceros national parks
Rhinoceros in south Nepal 3588 160 vulnerable
unicornis and north India
Javan rhinoceros one national
Rhinoceros park in Java, 72 0 critically endangered
sondaicus Indonesia
Sumatran national parks
rhinoceros in Sumatra
<80 7 critically endangered
Dicerorhinus and Borneo in
sumatrensis Indonesia

© UCLES 2021 0610/43/M/J/21


13

(a) Outline the reasons why large mammals, such as some species of rhinoceros, are
endangered.

...................................................................................................................................................

...................................................................................................................................................

...................................................................................................................................................

...................................................................................................................................................

...................................................................................................................................................

...................................................................................................................................................

...................................................................................................................................................

...................................................................................................................................................

...................................................................................................................................................

...................................................................................................................................................

............................................................................................................................................. [5]

(b) In 2013, Cincinnati Zoo in the USA announced that they were hoping to breed together
rhinoceros that were brother and sister as part of their captive breeding programme.

(i) Discuss the problems that using such closely related animals might have for the
conservation of rhinoceros.

...........................................................................................................................................

...........................................................................................................................................

...........................................................................................................................................

...........................................................................................................................................

...........................................................................................................................................

..................................................................................................................................... [2]

© UCLES 2021 0610/43/M/J/21 [Turn over


14

(ii) Captive breeding programmes often use artificial insemination.

Outline the process of artificial insemination.

...........................................................................................................................................

...........................................................................................................................................

...........................................................................................................................................

...........................................................................................................................................

...........................................................................................................................................

...........................................................................................................................................

...........................................................................................................................................

..................................................................................................................................... [3]

[Total: 10]

© UCLES 2021 0610/43/M/J/21


15

5 (a) Table 5.1 shows four functions of the human gas exchange system.

Complete Table 5.1 by identifying the part of the human gas exchange system that carries out
each function.

Table 5.1

part of the human gas exchange


function
system

contracts to decrease air pressure in the


lungs

allows air to flow between the larynx and


the lungs

contracts to lower the ribs in forced


expiration

site of gas exchange

[4]

© UCLES 2021 0610/43/M/J/21 [Turn over


16

(b) Scientists investigated the effect of increasing carbon dioxide concentration in the air on
the rate and depth of breathing. The people who took part in the investigation were healthy
volunteers.

The people breathed atmospheric air containing 0.04% carbon dioxide for five minutes.
They then breathed air containing 2% carbon dioxide for five minutes and then returned to
breathing atmospheric air for a further five minutes.

The results are shown in Fig. 5.1.

0.04% CO2 2% CO2 0.04% CO2

70

60

50

40

percentage 30
change in rate
and depth of
breathing 20

10

0
0 2 4 6 8 10 12 14 16

–10

–20
time / minutes
key:
rate of breathing
depth of breathing

Fig. 5.1

© UCLES 2021 0610/43/M/J/21


17

Describe the effects of changing the carbon dioxide concentration of the air on the rate and
depth of breathing, as shown in Fig. 5.1.

...................................................................................................................................................

...................................................................................................................................................

...................................................................................................................................................

...................................................................................................................................................

...................................................................................................................................................

...................................................................................................................................................

...................................................................................................................................................

...................................................................................................................................................

............................................................................................................................................. [4]

(c) Complete the sentences by writing the appropriate word or phrase in the spaces provided.

During physical activity the carbon dioxide concentration in the blood increases. The

reason for this is an increase in the rate of ............................................... that occurs in the

............................................... in muscle cells.

The increase in the concentration of carbon dioxide in the blood is detected by the

............................................... , which increases the rate and depth of breathing so that carbon

dioxide can be ............................................... from the body. This is important because carbon

dioxide is a ............................................... substance and can cause harm if it accumulates.


[5]

[Total: 13]

© UCLES 2021 0610/43/M/J/21 [Turn over


18

6 There are four blood groups in the ABO system in humans: A, B, AB and O.

The gene that determines blood group has three alleles: IA, IB and Io.

(a) Parents with the genotypes IAIo and IBIo are planning to have more children.

Complete the genetic diagram to determine the probability that the next child will have blood
group O.

parental blood groups A B

parental genotypes IAIo IBIo

Punnett square

phenotypes of
the children ...............................................................................................................................

probability that the child will have blood group O .....................................................................


[4]

(b) Explain why the ABO blood group system is an example of co-dominance.

...................................................................................................................................................

...................................................................................................................................................

...................................................................................................................................................

...................................................................................................................................................

............................................................................................................................................. [2]

© UCLES 2021 0610/43/M/J/21


19

(c) Fig. 6.1 shows the percentages of the global population with the four different blood groups in
the ABO system.

45

40

35

30

percentage 25
of the global
population
20

15

10

0
A B AB O
blood group

Fig. 6.1

With reference to Fig. 6.1, explain why the ABO blood group system is an example of
discontinuous variation.

...................................................................................................................................................

...................................................................................................................................................

...................................................................................................................................................

...................................................................................................................................................

...................................................................................................................................................

............................................................................................................................................. [2]

[Total: 8]

© UCLES 2021 0610/43/M/J/21


Cambridge IGCSE™
* 8 9 4 8 6 9 2 8 8 4 *

BIOLOGY 0610/43
Paper 4 Theory (Extended) May/June 2022

1 hour 15 minutes

You must answer on the question paper.

No additional materials are needed.

INSTRUCTIONS
● Answer all questions.
● Use a black or dark blue pen. You may use an HB pencil for any diagrams or graphs.
● Write your name, centre number and candidate number in the boxes at the top of the page.
● Write your answer to each question in the space provided.
● Do not use an erasable pen or correction fluid.
● Do not write on any bar codes.
● You may use a calculator.
● You should show all your working and use appropriate units.

INFORMATION
● The total mark for this paper is 80.
● The number of marks for each question or part question is shown in brackets [ ].

This document has 24 pages. Any blank pages are indicated.

DC (CE/CT) 303967/4
© UCLES 2022 [Turn over
2

1 Penicillin is produced by biotechnology industries.

(a) (i) State the name of the type of pathogen penicillin is used to treat.

..................................................................................................................................... [1]

(ii) State the name of the group of medicinal drugs that includes penicillin.

..................................................................................................................................... [1]

(b) Fig. 1.1 is a flow diagram of some of the steps in the production of penicillin.

organism A in step 4
a test-tube

X Y Z
step 5
gas outflow
pipe

step 1 filter

step 3
nutrient
mixing tank step 8
step 6
sterilising tank penicillin

step 2 step 7
fermenter not to scale

Fig. 1.1

(i) Organism A belongs to the fungus kingdom.

State two main features of fungal cells that are used to distinguish them from the cells of
prokaryotes.

1 ........................................................................................................................................

2 ........................................................................................................................................
[2]

(ii) State the genus name of organism A in Fig. 1.1.

..................................................................................................................................... [1]

© UCLES 2022 0610/43/M/J/22


3

(iii) Penicillin is produced in the fermenter shown in Fig. 1.1. A variety of nutrients, X, Y and Z,
are mixed together and added to the fermenter in step 1.

List two nutrients that need to be added to a fermenter to produce penicillin.

1 ........................................................................................................................................

2 ........................................................................................................................................
[2]

(iv) Explain why the nutrients are sterilised (step 2) before they are added to the fermenter
(step 3).

...........................................................................................................................................

...........................................................................................................................................

..................................................................................................................................... [1]

(v) Explain why the fermenter has a gas outflow pipe.

...........................................................................................................................................

...........................................................................................................................................

...........................................................................................................................................

...........................................................................................................................................

..................................................................................................................................... [2]

(vi) Using the information in Fig. 1.1, outline the events occurring from step 4 to step 8 during
the production of penicillin.

...........................................................................................................................................

...........................................................................................................................................

...........................................................................................................................................

...........................................................................................................................................

...........................................................................................................................................

...........................................................................................................................................

...........................................................................................................................................

...........................................................................................................................................

..................................................................................................................................... [4]

[Total: 14]

© UCLES 2022 0610/43/M/J/22 [Turn over


4

2 Red blood cells contain the protein haemoglobin.

(a) (i) State the names of the four chemical elements that are found in all proteins.

...........................................................................................................................................

..................................................................................................................................... [2]

(ii) State the role of haemoglobin.

...........................................................................................................................................

...........................................................................................................................................

..................................................................................................................................... [1]

(b) Fig. 2.1 shows a photomicrograph of some red blood cells from a person with sickle cell
anaemia.

Abnormal red blood cells occur because of a mutation in the gene for haemoglobin.

abnormal red
blood cell

Fig. 2.1

Suggest how the shape of the abnormal red blood cell shown in Fig. 2.1 will affect blood flow.

...................................................................................................................................................

...................................................................................................................................................

...................................................................................................................................................

...................................................................................................................................................

............................................................................................................................................. [2]

© UCLES 2022 0610/43/M/J/22


5

(c) The allele for the normal form of haemoglobin is HbA.

The allele for the abnormal form of haemoglobin is HbS.

Draw a genetic diagram to determine the probability of two heterozygous parents having a
child who does not have the HbS allele.

parental phenotypes ................................ x ................................

parental genotypes ................................ x ................................

gametes , x ,
............. ............. ............. .............

offspring genotypes ..................................................................................................................

probability of offspring not having the HbS allele .....................................................................


[5]

© UCLES 2022 0610/43/M/J/22 [Turn over


6

(d) Fig. 2.2 and Fig. 2.3 are maps showing some of the different regions in a country. Scientists
studied the distribution of the HbS allele in the country.

Fig. 2.2 shows the estimated frequency of the allele within the population.

Fig. 2.3 shows the estimated number of babies born with sickle cell anaemia in each region.

low

estimated frequency A
of the HbS allele C
in the population
B

high

Fig. 2.2

estimated number A
low
of babies born C
medium
with sickle cell
anaemia high B

Fig. 2.3

© UCLES 2022 0610/43/M/J/22


7

The scientists made a statement:

‘There is a relationship between the frequency of the HbS allele and the number of babies
born with sickle cell anaemia in regions A, B and C.’

(i) Using the information in Fig. 2.2 and Fig. 2.3, discuss the evidence for and against this
statement for regions A, B and C only.

...........................................................................................................................................

...........................................................................................................................................

...........................................................................................................................................

...........................................................................................................................................

...........................................................................................................................................

...........................................................................................................................................

...........................................................................................................................................

...........................................................................................................................................

...........................................................................................................................................

...........................................................................................................................................

..................................................................................................................................... [5]

(ii) Suggest how the scientists would identify the presence of the HbS allele in tissue
samples.

...........................................................................................................................................

...........................................................................................................................................

..................................................................................................................................... [1]

© UCLES 2022 0610/43/M/J/22 [Turn over


8

(e) Mutations are always inherited in single-celled organisms that reproduce asexually but are
not always inherited in organisms that reproduce sexually.

Explain why.

...................................................................................................................................................

...................................................................................................................................................

...................................................................................................................................................

...................................................................................................................................................

...................................................................................................................................................

...................................................................................................................................................

...................................................................................................................................................

...................................................................................................................................................

............................................................................................................................................. [4]

[Total: 20]

© UCLES 2022 0610/43/M/J/22


9

3 Acid rain has destroyed many forests including the forest shown in Fig. 3.1.

Fig. 3.1

(a) Describe how acid rain destroys forests.

...................................................................................................................................................

...................................................................................................................................................

...................................................................................................................................................

...................................................................................................................................................

...................................................................................................................................................

...................................................................................................................................................

............................................................................................................................................. [3]

© UCLES 2022 0610/43/M/J/22 [Turn over


10

(b) Explain the negative consequences to the environment of destroying forests.

...................................................................................................................................................

...................................................................................................................................................

...................................................................................................................................................

...................................................................................................................................................

...................................................................................................................................................

...................................................................................................................................................

...................................................................................................................................................

...................................................................................................................................................

............................................................................................................................................. [4]

(c) Acid rain can also damage aquatic organisms such as the amphibian shown in Fig. 3.2.

Fig. 3.2

Suggest why amphibians are vulnerable to pollutants such as acid rain.

...................................................................................................................................................

...................................................................................................................................................

...................................................................................................................................................

...................................................................................................................................................

............................................................................................................................................. [2]

© UCLES 2022 0610/43/M/J/22


11

(d) Many countries have strict laws to prevent acid rain.

Describe how countries have reduced acid rain.

...................................................................................................................................................

...................................................................................................................................................

...................................................................................................................................................

...................................................................................................................................................

...................................................................................................................................................

...................................................................................................................................................

............................................................................................................................................. [3]

[Total: 12]

© UCLES 2022 0610/43/M/J/22 [Turn over


12

4 (a) Fig. 4.1 is a diagram of the alimentary canal.

Fig. 4.1

© UCLES 2022 0610/43/M/J/22


13

Complete Table 4.1 by stating:

• the names of the organs from Fig. 4.1


• the letters of all the processes shown in the key that occur in each organ.

Key:
A – absorption
C – chemical digestion
E – egestion
I – ingestion
M – mechanical digestion

Table 4.1

number from letter or letters of all the


name of the organ
Fig. 4.1 processes that occur in the organ

[6]

© UCLES 2022 0610/43/M/J/22 [Turn over


14

(b) Fig. 4.2 is a diagram of a villus. The arrow indicates the direction of blood flow.

U S
T

not to scale

Fig. 4.2

© UCLES 2022 0610/43/M/J/22


15

Describe the structure of a villus and its role in the alimentary canal.

Use the letters in Fig. 4.2 to support your answer.

...................................................................................................................................................

...................................................................................................................................................

...................................................................................................................................................

...................................................................................................................................................

...................................................................................................................................................

...................................................................................................................................................

...................................................................................................................................................

...................................................................................................................................................

...................................................................................................................................................

...................................................................................................................................................

...................................................................................................................................................

...................................................................................................................................................

............................................................................................................................................. [6]

[Total: 12]

© UCLES 2022 0610/43/M/J/22 [Turn over


16

5 Glasshouses are designed to maximise crop plant yield.

(a) (i) Explain why carbon dioxide enrichment is used in many glasshouses to increase crop
plant yield.

...........................................................................................................................................

...........................................................................................................................................

...........................................................................................................................................

...........................................................................................................................................

...........................................................................................................................................

...........................................................................................................................................

..................................................................................................................................... [3]

(ii) Suggest how the carbon dioxide concentration in a glasshouse can be enriched.

...........................................................................................................................................

...........................................................................................................................................

..................................................................................................................................... [1]

(iii) Outline how carbon dioxide in a glasshouse moves into leaves.

...........................................................................................................................................

...........................................................................................................................................

...........................................................................................................................................

...........................................................................................................................................

..................................................................................................................................... [2]

© UCLES 2022 0610/43/M/J/22


17

(b) Additional lighting is often installed in glasshouses in countries with temperate climates.

Table 5.1 summarises some of the factors that are considered by plant growers when
choosing the type of lamps to install in a glasshouse.

Table 5.1

type of electrical energy used light intensity output


notes
lamp by the lamp / J per s / arbitrary units
• releases lots of heat
sodium 1041 1767 • best when used in
addition to sunlight
• releases very little heat
LED 423 378 • can be used as an
alternative to sunlight
• releases some heat
metal
651 817 • can be used as an
halide
alternative to sunlight
• releases some heat
fluorescent 394 374 • best when used in
addition to sunlight

(i) Calculate the percentage increase in the energy used by the metal halide lamp compared
to the energy used by the fluorescent lamp.

Give your answer to two significant figures.

Space for working.

............................................................ %
[2]

(ii) State which type of lamp has the highest light intensity output per unit of electrical energy
used.

..................................................................................................................................... [1]

© UCLES 2022 0610/43/M/J/22 [Turn over


18

(iii) Some types of lamp release a lot of heat.

Explain the possible effects of excessive heat on the plants in a glasshouse.

...........................................................................................................................................

...........................................................................................................................................

...........................................................................................................................................

...........................................................................................................................................

...........................................................................................................................................

...........................................................................................................................................

..................................................................................................................................... [3]

[Total: 12]

© UCLES 2022 0610/43/M/J/22


19

6 (a) Fig. 6.1 shows a photograph of fruit attached to the branch of an orange tree, Citrus sinensis.

Fig. 6.1

State one reason why orange trees are classified as dicotyledonous plants.

...................................................................................................................................................

...................................................................................................................................................

............................................................................................................................................. [1]

(b) Biotechnologists use enzymes to extract juice from fruit such as oranges.

Define the term enzyme.

...................................................................................................................................................

...................................................................................................................................................

...................................................................................................................................................

...................................................................................................................................................

............................................................................................................................................. [2]

© UCLES 2022 0610/43/M/J/22 [Turn over


20

(c) Fig. 6.2 shows the results of leaving pieces of orange fruit in an enzyme solution for different
lengths of time.

total volume
7
of juice
extracted
/ cm3 6

4
0 20 40 60 80 100
time / minutes

Fig. 6.2

(i) State the name of the enzyme used to extract juice from fruit.

..................................................................................................................................... [1]

(ii) Using the information in Fig. 6.2, state the optimum length of time for efficient extraction
of juice from oranges.

..................................................................................................................................... [1]

(iii) State the name of the vitamin found in high concentrations in citrus fruit such as oranges.

..................................................................................................................................... [1]

(d) The genes in some plants have been changed to increase the concentration of vitamins that
these plants produce.

(i) State the name of the process of changing the genes of a plant.

..................................................................................................................................... [1]

© UCLES 2022 0610/43/M/J/22


21

(ii) Discuss the possible disadvantages of people changing the genes in a plant.

...........................................................................................................................................

...........................................................................................................................................

...........................................................................................................................................

...........................................................................................................................................

...........................................................................................................................................

...........................................................................................................................................

..................................................................................................................................... [3]

[Total: 10]

© UCLES 2022 0610/43/M/J/22


Cambridge IGCSE™
* 4 9 4 6 1 0 0 9 5 4 *

BIOLOGY 0610/43
Paper 4 Theory (Extended) May/June 2023

1 hour 15 minutes

You must answer on the question paper.

No additional materials are needed.

INSTRUCTIONS
● Answer all questions.
● Use a black or dark blue pen. You may use an HB pencil for any diagrams or graphs.
● Write your name, centre number and candidate number in the boxes at the top of the page.
● Write your answer to each question in the space provided.
● Do not use an erasable pen or correction fluid.
● Do not write on any bar codes.
● You may use a calculator.
● You should show all your working and use appropriate units.

INFORMATION
● The total mark for this paper is 80.
● The number of marks for each question or part question is shown in brackets [ ].

This document has 20 pages. Any blank pages are indicated.

DC (KN/FC) 311949/4
© UCLES 2023 [Turn over
2

1 (a) The movement of molecules within an organism can occur by diffusion and active transport.

Complete Table 1.1 by placing ticks (3) to show the correct features of each process.

Table 1.1

active
feature diffusion
transport

movement of particles always occurs across a cell membrane

movement of molecules during gas exchange

rate of movement of particles is higher when the concentration


gradient is larger

requires energy from respiration

[4]

(b) Explain why active transport is important in root hair cells.

...................................................................................................................................................

...................................................................................................................................................

...................................................................................................................................................

...................................................................................................................................................

...................................................................................................................................................

...................................................................................................................................................

............................................................................................................................................. [3]

[Total: 7]

© UCLES 2023 0610/43/M/J/23


3

2 Transpiration is the loss of water vapour from the leaves of a plant.

(a) Complete the sentence describing transpiration.

Water evaporates from the surfaces of the ............................................... cells into the air

spaces and then ............................................... out of the leaves, through the stomata, as

water vapour.
[2]

(b) Explain how water moves upwards in the xylem.

...................................................................................................................................................

...................................................................................................................................................

...................................................................................................................................................

...................................................................................................................................................

...................................................................................................................................................

...................................................................................................................................................

............................................................................................................................................. [3]

© UCLES 2023 0610/43/M/J/23 [Turn over


4

(c) A student investigated the effect of wind speed on the rate of transpiration in a small
Pachira aquatica tree.

They measured the rate of transpiration when the plant was placed in different wind speeds
in both the light and the dark. The tree was given an adequate supply of water.

The results of this investigation are shown in Fig. 2.1.

rate of 4
transpiration
/ g per hr 3

0
0 2 4 6 8 10 12

wind speed / m per s

light dark

Fig. 2.1

(i) Describe and explain the results shown in Fig. 2.1.

...........................................................................................................................................

...........................................................................................................................................

...........................................................................................................................................

...........................................................................................................................................

...........................................................................................................................................

...........................................................................................................................................

...........................................................................................................................................

...........................................................................................................................................

..................................................................................................................................... [4]

© UCLES 2023 0610/43/M/J/23


5

(ii) The investigation was repeated in the light with the same species of tree in an
environment where the humidity was higher.

Draw a line on Fig. 2.1 to predict the results in the light with increased humidity. [1]

(iii) The investigation described in 2(c) was repeated (different wind speeds in both the light
and the dark).

However, the tree was not given an adequate supply of water during the investigation.

Predict the result and explain your prediction.

...........................................................................................................................................

...........................................................................................................................................

...........................................................................................................................................

...........................................................................................................................................

...........................................................................................................................................

...........................................................................................................................................

...........................................................................................................................................

...........................................................................................................................................

..................................................................................................................................... [4]

[Total: 14]

© UCLES 2023 0610/43/M/J/23 [Turn over


6

3 Fig. 3.1 is a diagram of a section through a human heart.

Fig. 3.1

(a) Draw an X on the septum in Fig. 3.1. [1]

(b) Explain the reason for the difference between the thickness of the walls at B and at C in
Fig. 3.1.

...................................................................................................................................................

...................................................................................................................................................

...................................................................................................................................................

...................................................................................................................................................

............................................................................................................................................. [2]

© UCLES 2023 0610/43/M/J/23


7

(c) A red blood cell enters the vena cava at A in Fig. 3.1.

Explain how the red blood cell is moved from the vena cava to the aorta.

...................................................................................................................................................

...................................................................................................................................................

...................................................................................................................................................

...................................................................................................................................................

...................................................................................................................................................

...................................................................................................................................................

...................................................................................................................................................

...................................................................................................................................................

...................................................................................................................................................

...................................................................................................................................................

...................................................................................................................................................

...................................................................................................................................................

............................................................................................................................................. [6]

© UCLES 2023 0610/43/M/J/23 [Turn over


8

(d) An athlete measured her heart rate during a running race. She recorded it before the race,
during the race and during her recovery.

Her results are shown in Fig. 3.2.

200

180

160

140
heart rate
/ beats per 120
minute
100

80

60

40
0 5 10 15 20
time / minutes
start end
of race of race

Fig. 3.2

(i) Suggest how the athlete could monitor the activity of her heart.

...........................................................................................................................................

...........................................................................................................................................

..................................................................................................................................... [1]

(ii) Calculate the percentage change in heart rate from the maximum heart rate reached
during the race until the heart rate recorded at 18 minutes.

Give your answer to three significant figures.

Space for working.

............................................................ %
[3]
© UCLES 2023 0610/43/M/J/23
9

(iii) Explain why heart rate must increase during exercise.

...........................................................................................................................................

...........................................................................................................................................

...........................................................................................................................................

...........................................................................................................................................

...........................................................................................................................................

...........................................................................................................................................

..................................................................................................................................... [3]

[Total: 16]

© UCLES 2023 0610/43/M/J/23 [Turn over


10

4 (a) Urea is a waste product.

(i) Describe how urea is formed.

...........................................................................................................................................

...........................................................................................................................................

...........................................................................................................................................

...........................................................................................................................................

...........................................................................................................................................

..................................................................................................................................... [3]

(ii) State the component of blood that transports urea.

..................................................................................................................................... [1]

(iii) State why urea must be excreted.

...........................................................................................................................................

..................................................................................................................................... [1]

(iv) State the name of the blood vessel that carries blood away from the kidney.

..................................................................................................................................... [1]

(b) Fig. 4.1 shows a drawing of a nephron in the human kidney and associated blood vessels.

K
L

Fig. 4.1

© UCLES 2023 0610/43/M/J/23


11

Describe how the structures labelled in Fig. 4.1 produce urine.

...................................................................................................................................................

...................................................................................................................................................

...................................................................................................................................................

...................................................................................................................................................

...................................................................................................................................................

...................................................................................................................................................

...................................................................................................................................................

...................................................................................................................................................

............................................................................................................................................. [4]

(c) Urea can be used as a fertiliser as it is a source of nitrogen.

Explain the importance of nitrate ions to plants.

...................................................................................................................................................

...................................................................................................................................................

............................................................................................................................................. [1]

(d) A farmer applied fertiliser to a field next to a lake.

Suggest two precautions the farmer should take when applying fertiliser to reduce the risk of
eutrophication occurring in the lake.

1 ................................................................................................................................................

2 ................................................................................................................................................
[2]

[Total: 13]

© UCLES 2023 0610/43/M/J/23 [Turn over


12

5 (a) Describe the role of progesterone in pregnancy.

...................................................................................................................................................

...................................................................................................................................................

...................................................................................................................................................

...................................................................................................................................................

...................................................................................................................................................

...................................................................................................................................................

............................................................................................................................................. [3]

(b) Fig. 5.1 shows a drawing of a section of a human placenta. The arrows show the direction of
blood flow.

maternal artery
from mother
to mother maternal vein

space filled
with maternal fetal capillary
blood

umbilical
arteries

from fetus
to fetus
from fetus umbilical vein
not to scale

Fig. 5.1

© UCLES 2023 0610/43/M/J/23


13

Substances move across the placenta by diffusion.

(i) State the names of two useful substances that move from the blood of the mother to the
blood of the fetus.

1 ........................................................................................................................................

2 ........................................................................................................................................
[2]

(ii) Using the information in Fig. 5.1, suggest how the placenta is adapted for efficient
diffusion.

...........................................................................................................................................

...........................................................................................................................................

...........................................................................................................................................

...........................................................................................................................................

...........................................................................................................................................

...........................................................................................................................................

..................................................................................................................................... [3]

(c) The Rhesus (Rh) factor is a protein that is found on the surface of red blood cells in some
people. If the protein is present then the individual is Rh positive.

The allele for the presence of the Rh factor is dominant and is represented by the letter D.
The recessive allele is represented by the letter d.

If a mother is Rh negative and the fetus is Rh positive there can be problems during pregnancy.

A Rh negative mother and a heterozygous Rh positive father have a child.

Complete the Punnett square and determine the probability of the child being Rh positive.

father’s gametes

mother’s gametes

probability .................................................................................................................................
[3]

[Total: 11]
© UCLES 2023 0610/43/M/J/23 [Turn over
14

6 (a) Fig. 6.1 shows the percentage of land covered by forest in a country from 1660 until 2000.

30

28

26

24

22
percentage of
land covered 20
by forest
18

16

14

12

10
1650 1750 1850 1950 2050
year

Fig. 6.1

(i) Using the information in Fig. 6.1, state which 10-year period had the highest increase in
the percentage of land covered by forest.

..................................................................................................................................... [1]

(ii) Outline how human activities could cause the change in the percentage of land covered
by forest from 1660 to 1800 shown in Fig. 6.1.

...........................................................................................................................................

...........................................................................................................................................

...........................................................................................................................................

...........................................................................................................................................

...........................................................................................................................................

...........................................................................................................................................

..................................................................................................................................... [3]

© UCLES 2023 0610/43/M/J/23


15

(b) It is estimated that 40% of plant species are at risk of extinction.

Seed banks can store seeds from many species for a long time.

Table 6.1 shows some information about one plant species that is stored in a seed bank.

Table 6.1

mass of one seed / g percentage oil content percentage protein content


1.96 71 11

(i) Using the data shown in Table 6.1, calculate the mass of protein in one seed.

....................................................... g [1]

(ii) Suggest why many plants can easily be conserved using seed banks.

...........................................................................................................................................

...........................................................................................................................................

...........................................................................................................................................

...........................................................................................................................................

..................................................................................................................................... [2]

© UCLES 2023 0610/43/M/J/23 [Turn over


16

(iii) Describe why conservation projects such as seed banks are important.

...........................................................................................................................................

...........................................................................................................................................

...........................................................................................................................................

...........................................................................................................................................

...........................................................................................................................................

...........................................................................................................................................

...........................................................................................................................................

...........................................................................................................................................

...........................................................................................................................................

...........................................................................................................................................

..................................................................................................................................... [5]

[Total: 12]

© UCLES 2023 0610/43/M/J/23


17

7 (a) State the name of a chemical element that is found in all proteins but not in carbohydrates or
fats.

............................................................................................................................................. [1]

(b) Table 7.1 shows the names of some biological molecules, enzymes and the organs that
produce the enzymes.

Complete Table 7.1.

Table 7.1

enzyme that
products of the
catalyses the
large biological breakdown of the organ that produces
breakdown of the
molecule large biological the enzyme
large biological
molecule
molecule

oil pancreas

glycogen
glycogen liver
phosphorylase

maltose amylase

protein stomach

[4]

(c) State two hormones that can affect the concentration of glycogen in the liver.

1 ................................................................................................................................................

2 ................................................................................................................................................
[2]

[Total: 7]

© UCLES 2023 0610/43/M/J/23


Cambridge International Examinations
Cambridge International General Certificate of Secondary Education
* 4 8 6 2 9 4 4 5 3 1 *

BIOLOGY 0610/43
Paper 4 Theory (Extended) October/November 2017
1 hour 15 minutes
Candidates answer on the Question Paper.
No Additional Materials are required.

READ THESE INSTRUCTIONS FIRST

Write your Centre number, candidate number and name on all the work you hand in.
Write in dark blue or black pen.
You may use an HB pencil for any diagrams or graphs.
Do not use staples, paper clips, glue or correction fluid.
DO NOT WRITE IN ANY BARCODES.

Answer all questions.

Electronic calculators may be used.


You may lose marks if you do not show your working or if you do not use appropriate units.

At the end of the examination, fasten all your work securely together.
The number of marks is given in brackets [ ] at the end of each question or part question.

This syllabus is approved for use in England, Wales and Northern Ireland as a Cambridge International Level 1/Level 2 Certificate.

This document consists of 18 printed pages and 2 blank pages.

DC (ST/SG) 133511/4
© UCLES 2017 [Turn over
2

1 Fig. 1.1 shows two photomicrographs of a cross-section of trachea.

low magnification high magnification

Fig. 1.1

(a) The open space at the centre of the trachea is labelled A in Fig. 1.1.

Air travels into and out of the trachea through this open space.

State a gas that is at a higher concentration in expired air than in inspired air.

...............................................................................................................................................[1]

(b) Describe and explain the functions of the structures or substances labelled B to E in Fig. 1.1.

Use the letters in Fig. 1.1 in your answer.

...................................................................................................................................................

...................................................................................................................................................

...................................................................................................................................................

...................................................................................................................................................

...................................................................................................................................................

...................................................................................................................................................

...................................................................................................................................................

...................................................................................................................................................

...............................................................................................................................................[4]

© UCLES 2017 0610/43/O/N/17


3

(c) Fig. 1.2 shows some events during inspiration.

P pressure in the thorax decreases


Q air travels down the trachea
R air enters the bronchi
S air travels through the larynx
T air enters the nose
U the ribcage moves upwards and outwards
V air enters the alveoli

Fig. 1.2

(i) Put the events shown in Fig. 1.2 into the correct sequence. Two have been done for you.

T V

[2]
(ii) Suggest why alveoli have thin walls.

...........................................................................................................................................

...........................................................................................................................................

...........................................................................................................................................

.......................................................................................................................................[2]

(d) Sickle-cell anaemia is a disease that reduces the delivery of oxygen to tissues.

Explain why.

...................................................................................................................................................

...................................................................................................................................................

...................................................................................................................................................

...................................................................................................................................................

...................................................................................................................................................

...................................................................................................................................................

...............................................................................................................................................[3]

[Total: 12]

© UCLES 2017 0610/43/O/N/17 [Turn over


5

2 A group of students investigated the effect of exercise on their heart rates.

They measured their heart rates:

• before exercise
• immediately after running 1 km
• one minute after running 1 km

Before doing the investigation they wrote a hypothesis.

(a) (i) Write a hypothesis for this investigation.

...........................................................................................................................................

...........................................................................................................................................

...........................................................................................................................................

.......................................................................................................................................[2]

(ii) The students measured their pulse as an indicator of heart rate.

Describe how the students could measure their pulse.

...........................................................................................................................................

...........................................................................................................................................

...........................................................................................................................................

.......................................................................................................................................[2]

(b) In another investigation, a doctor tested some of her patients to determine the effect of
exercise on coronary heart disease.

Coronary heart disease is caused by a blockage in the coronary artery.

Describe the effect on the heart of a blockage in the coronary artery.

...................................................................................................................................................

...................................................................................................................................................

...................................................................................................................................................

...................................................................................................................................................

...............................................................................................................................................[2]

© UCLES 2017 0610/43/O/N/17 [Turn over


6

(c) The doctor divided her coronary heart disease patients randomly into two equal groups.

Each group was given different instructions:

• group A – patients were given a daily exercise plan


• group B – patients were told to make their own exercise plan.

The doctor measured the heart rate (HR) of each patient immediately after doing exercise
and again one minute later.

She calculated their heart rate recovery using this formula:

heart rate recovery = HR immediately after exercise – HR one minute after exercise.

She then calculated the average heart rate recovery for each of the two groups of patients.

The doctor repeated these measurements after three months and after six months.

The results are shown in Fig. 2.1.

30
Key:
high group A
25
fitness
group B
20
average
heart rate recovery medium
15
/ beats per minute fitness

10
low
5
fitness

0
0 3 6
time / months

Fig. 2.1

© UCLES 2017 0610/43/O/N/17


7

Describe and explain the effect of exercise on the average heart rate recovery of the coronary
heart disease patients in group A and group B.

Use data from Fig. 2.1 to support your answer.

...................................................................................................................................................

...................................................................................................................................................

...................................................................................................................................................

...................................................................................................................................................

...................................................................................................................................................

...................................................................................................................................................

...................................................................................................................................................

...................................................................................................................................................

...................................................................................................................................................

...................................................................................................................................................

...................................................................................................................................................

...................................................................................................................................................

...............................................................................................................................................[6]

(d) Exercise may reduce the risk of coronary heart disease.

State one other possible way of reducing the risk of developing coronary heart disease.

...............................................................................................................................................[1]

[Total: 13]

© UCLES 2017 0610/43/O/N/17 [Turn over


8

3 Apple scab is a disease that infects apple trees.

Fig. 3.1 shows apples from uninfected and infected apple trees.

uninfected apple tree infected apple tree

Fig. 3.1

There is a gene that determines whether or not apple trees are resistant to apple scab disease.

There are two alleles for this gene:

• disease-resistant, R
• not disease-resistant, r

(a) (i) Complete the sentence.

Genes and alleles are made of ..................................................... . [1]

© UCLES 2017 0610/43/O/N/17


9

(ii) A farmer wanted to do a test cross to identify the genotype of disease-resistant apple
trees. This would tell him whether his trees were either homozygous dominant or
heterozygous.

Determine the phenotypes of the offspring if the unknown parent apple tree was
heterozygous.

Complete the genetic diagram:

parental phenotypes disease-resistant × not disease-resistant

parental genotypes .................................. × ..................................

gametes ......... ......... × ......... .........


, ,

offspring genotype ....................................................................................

offspring phenotype ....................................................................................


[5]

(b) The farmer wanted to breed disease-resistant apple trees.

(i) He decided not to use heterozygous disease-resistant apple trees in his selective
breeding programme.

Explain why.

...........................................................................................................................................

...........................................................................................................................................

...........................................................................................................................................

...........................................................................................................................................

...........................................................................................................................................

...........................................................................................................................................

.......................................................................................................................................[2]

© UCLES 2017 0610/43/O/N/17 [Turn over


10

(ii) The farmer wanted to be sure that only the selected disease-resistant apple trees would
reproduce.

Suggest what the farmer could do to ensure that only the selected apple trees were
pollinated.

...........................................................................................................................................

...........................................................................................................................................

.......................................................................................................................................[1]

(iii) Describe how artificial selection differs from natural selection.

...........................................................................................................................................

...........................................................................................................................................

...........................................................................................................................................

...........................................................................................................................................

.......................................................................................................................................[2]

[Total: 11]

© UCLES 2017 0610/43/O/N/17


12

4 The Canadian Government were concerned about overfishing at the Grand Banks in the Atlantic
Ocean.

As a result, commercial fish stocks were monitored from 2002 until 2013.

The population data for four species of fish are shown in Fig. 4.1.

140 species K
120
100
number of fish 80
/ million 60
40
20
0
2002 2003 2004 2005 2006 2007 2008 2009 2010 2011 2012 2013
year

700 species L
600
500
number of fish 400
/ million 300
200
100
0
2002 2003 2004 2005 2006 2007 2008 2009 2010 2011 2012 2013
year

700 species M
600
500
number of fish 400
/ million 300
200
100
0
2002 2003 2004 2005 2006 2007 2008 2009 2010 2011 2012 2013
year
2500
species N
2000

number of fish 1500


/ million
1000

500

0
2002 2003 2004 2005 2006 2007 2008 2009 2010 2011 2012 2013
year
Fig. 4.1
© UCLES 2017 0610/43/O/N/17
13

(a) Use the information in Fig. 4.1 to:

(i) state the most abundant fish species in 2002

.......................................................................................................................................[1]

(ii) suggest the fish species that had the most carefully controlled fishing quotas between
2002 and 2013.

Give a reason for your choice.

...........................................................................................................................................

...........................................................................................................................................

.......................................................................................................................................[1]

(iii) calculate the percentage increase in species N between 2002 and 2003.

Show your working.

................................................................
[2]

(b) Overfishing is a possible reason for the decrease of the population of species M between
2002 and 2003.

State two other reasons that could have caused this decrease.

1 ................................................................................................................................................

2 ................................................................................................................................................
[2]

© UCLES 2017 0610/43/O/N/17 [Turn over


14

(c) Overfishing can be reduced by having large holes in fishing nets.

Fig. 4.2 shows sections of two fishing nets, the drawings are both at the same scale.

fishing net with fishing net with


large holes small holes

Fig. 4.2

(i) Suggest how controlling the size of the holes in fishing nets helps to reduce overfishing.

...........................................................................................................................................

...........................................................................................................................................

.......................................................................................................................................[1]

(ii) Describe and explain how methods other than fishing net hole size, could help to prevent
overfishing.

...........................................................................................................................................

...........................................................................................................................................

...........................................................................................................................................

...........................................................................................................................................

...........................................................................................................................................

...........................................................................................................................................

...........................................................................................................................................

...........................................................................................................................................

.......................................................................................................................................[4]

© UCLES 2017 0610/43/O/N/17


15

(d) Fig. 4.3 shows part of a food web at the Grand Banks.

guillemot gull seal

cod squid

zooplankton

phytoplankton

Fig. 4.3

Cod is a species of fish that almost became extinct in the Grand Banks due to overfishing.

Suggest how the extinction of cod could affect the food web in Fig. 4.3.

...................................................................................................................................................

...................................................................................................................................................

...................................................................................................................................................

...................................................................................................................................................

...................................................................................................................................................

...................................................................................................................................................

...................................................................................................................................................

...................................................................................................................................................

...................................................................................................................................................

...................................................................................................................................................

...............................................................................................................................................[4]

© UCLES 2017 0610/43/O/N/17 [Turn over


16

(e) Sustainable development is required to manage fish stocks.

Define the term sustainable development.

...................................................................................................................................................

...................................................................................................................................................

...............................................................................................................................................[2]

[Total: 17]

5 The kingdom Fungi contains a great diversity of organisms including yeasts, moulds and
mushrooms.

Like plants, fungi contain nuclei and mitochondria.

(a) (i) State the function of mitochondria.

...........................................................................................................................................

.......................................................................................................................................[2]

(ii) State two characteristics of fungi that are used to distinguish them from plants.

1 ........................................................................................................................................

2 ........................................................................................................................................
[2]

(b) Yeast is a single-celled fungus that is used in bread-making.

Explain why yeast is used in bread-making.

...................................................................................................................................................

...................................................................................................................................................

...................................................................................................................................................

...................................................................................................................................................

...................................................................................................................................................

...................................................................................................................................................

...............................................................................................................................................[3]

© UCLES 2017 0610/43/O/N/17


17

(c) Penicillium is a mould fungus that is used to make antibiotics.

(i) Describe how Penicillium is used to make the antibiotic penicillin.

...........................................................................................................................................

...........................................................................................................................................

...........................................................................................................................................

...........................................................................................................................................

...........................................................................................................................................

...........................................................................................................................................

.......................................................................................................................................[3]

(ii) Explain why antibiotics can be used to treat bacterial infections but not viral infections.

...........................................................................................................................................

...........................................................................................................................................

.......................................................................................................................................[1]

(d) Some fungi are human pathogens.

Describe how the human body prevents pathogens from entering.

...................................................................................................................................................

...................................................................................................................................................

...................................................................................................................................................

...................................................................................................................................................

...................................................................................................................................................

...................................................................................................................................................

...............................................................................................................................................[3]

[Total: 14]

© UCLES 2017 0610/43/O/N/17 [Turn over


18

6 Fig. 6.1 is a flow chart of some of the events that occur to maintain a constant body temperature.

body temperature rises

temperature receptors
detect a stimulus

the central nervous system receives


an impulse and sends the impulse on

sweat glands respond


blood vessels respond
hair erector muscles respond

body temperature falls


to the set level

Fig. 6.1

© UCLES 2017 0610/43/O/N/17


19

(a) (i) State the names of the types of neurones at X and Y in Fig. 6.1.

X ........................................................................................................................................

Y ........................................................................................................................................
[2]

(ii) State the name of one effector shown in Fig. 6.1.

.......................................................................................................................................[1]

(iii) State the name of the mechanism that controls homeostasis which is represented by the
flowchart in Fig. 6.1.

.......................................................................................................................................[1]

(b) (i) Describe how shunt vessels in the skin function to help cool the body when the body
temperature is high.

...........................................................................................................................................

...........................................................................................................................................

...........................................................................................................................................

...........................................................................................................................................

...........................................................................................................................................

...........................................................................................................................................

.......................................................................................................................................[3]

(ii) Describe how the sweat glands and the hair erector muscles function in mammals when
the external environment is hot.

...........................................................................................................................................

...........................................................................................................................................

...........................................................................................................................................

...........................................................................................................................................

...........................................................................................................................................

...........................................................................................................................................

.......................................................................................................................................[3]

© UCLES 2017 0610/43/O/N/17 [Turn over


20

(c) (i) Suggest an advantage of using neurones rather than hormones to regulate body
temperature.

...........................................................................................................................................

...........................................................................................................................................

.......................................................................................................................................[1]

(ii) List two hormones that are involved in homeostasis.

1 ........................................................................................................................................

2 ........................................................................................................................................
[2]

[Total: 13]

Permission to reproduce items where third-party owned material protected by copyright is included has been sought and cleared where possible. Every
reasonable effort has been made by the publisher (UCLES) to trace copyright holders, but if any items requiring clearance have unwittingly been included, the
publisher will be pleased to make amends at the earliest possible opportunity.

To avoid the issue of disclosure of answer-related information to candidates, all copyright acknowledgements are reproduced online in the Cambridge International
Examinations Copyright Acknowledgements Booklet. This is produced for each series of examinations and is freely available to download at www.cie.org.uk after
the live examination series.

Cambridge International Examinations is part of the Cambridge Assessment Group. Cambridge Assessment is the brand name of University of Cambridge Local
Examinations Syndicate (UCLES), which is itself a department of the University of Cambridge.

© UCLES 2017 0610/43/O/N/17


Cambridge International Examinations
Cambridge International General Certificate of Secondary Education
* 2 4 8 0 3 9 0 3 5 7 *

BIOLOGY 0610/43
Paper 4 Theory (Extended) October/November 2018
1 hour 15 minutes
Candidates answer on the Question Paper.
No Additional Materials are required.

READ THESE INSTRUCTIONS FIRST

Write your Centre number, candidate number and name on all the work you hand in.
Write in dark blue or black pen.
You may use an HB pencil for any diagrams or graphs.
Do not use staples, paper clips, glue or correction fluid.
DO NOT WRITE IN ANY BARCODES.

Answer all questions.

Electronic calculators may be used.


You may lose marks if you do not show your working or if you do not use appropriate units.

At the end of the examination, fasten all your work securely together.
The number of marks is given in brackets [ ] at the end of each question or part question.

This syllabus is approved for use in England, Wales and Northern Ireland as a Cambridge International Level 1/Level 2 Certificate.

This document consists of 17 printed pages and 3 blank pages.

DC (CE/SW) 152729/4
© UCLES 2018 [Turn over
2

1 Water is a very important molecule for all living organisms.

(a) (i) State the name of the organ in plants where most water is absorbed.

.......................................................................................................................................[1]

(ii) State the name of the organ in humans where most water is absorbed.

.......................................................................................................................................[1]

(iii) State one property of water that makes it useful to animals and plants.

.......................................................................................................................................[1]

(b) The flow diagram in Fig. 1.1 shows a town and part of the water cycle.

F G K L M N
O
J

Fig. 1.1

© UCLES 2018 0610/43/O/N/18


3

Table 1.1 describes some of the processes in the water cycle.

Complete Table 1.1.

One row has been done for you.

Table 1.1

description name of the process letter in Fig. 1.1

nitrate ions are washed into rivers leaching F

an algal bloom in the water is caused by


leaching of nitrate ions

evaporation

conversion of water from a vapour to a


liquid

transpiration

[4]

(c) Polluted water can be purified at a sewage treatment works.

(i) State one reason why it is necessary to treat polluted water before it is used as drinking
water.

...........................................................................................................................................

.......................................................................................................................................[1]

(ii) Outline the process of sewage treatment. You may use the letters in Fig. 1.1 in your
answer.

...........................................................................................................................................

...........................................................................................................................................

...........................................................................................................................................

...........................................................................................................................................

...........................................................................................................................................

...........................................................................................................................................

...........................................................................................................................................

...........................................................................................................................................

.......................................................................................................................................[4]

[Total: 12]
© UCLES 2018 0610/43/O/N/18 [Turn over
4

2 The Indian muntjac deer, Muntiacus muntjak, is recorded as the mammal with the lowest number
of chromosomes.

Fig. 2.1 is an image of the chromosomes in the nucleus of a diploid cell of a female muntjac deer.

sex chromosomes

Fig. 2.1

(a) State the diploid number of chromosomes for the female muntjac deer.

................................................................... [1]

(b) Fig. 2.2 is an image of the chromosomes in the nucleus of a diploid cell of a male muntjac
deer.

sex chromosomes

Fig. 2.2

Describe how the sex chromosomes of the male muntjac deer shown in Fig. 2.2 differ from
those of the female shown in Fig. 2.1.

...................................................................................................................................................

...................................................................................................................................................

...............................................................................................................................................[2]

© UCLES 2018 0610/43/O/N/18


5

(c) Explain how meiosis can result in variation in a species.

Use the words chromosome and gametes in your answer.

...................................................................................................................................................

...................................................................................................................................................

...................................................................................................................................................

...................................................................................................................................................

...................................................................................................................................................

...................................................................................................................................................

...................................................................................................................................................

...................................................................................................................................................

...............................................................................................................................................[4]

(d) Another cause of variation is the formation of new alleles.

Describe how new alleles can be formed.

...................................................................................................................................................

...................................................................................................................................................

...................................................................................................................................................

...................................................................................................................................................

...................................................................................................................................................

...................................................................................................................................................

...............................................................................................................................................[3]

[Total: 10]

© UCLES 2018 0610/43/O/N/18 [Turn over


7

3 (a) Fig. 3.1 is a photomicrograph of part of the upper surface of a broad bean leaf, Vicia faba.

Fig. 3.1

(i) On Fig. 3.1, identify and label two structures that are visible in cell P. [2]

(ii) State the name of the tissue shown in Fig. 3.1.

.......................................................................................................................................[1]

(iii) The tissue shown in Fig. 3.1 is transparent.

Explain why it is important to the plant that the tissue shown in Fig. 3.1 is transparent.

...........................................................................................................................................

...........................................................................................................................................

...........................................................................................................................................

...........................................................................................................................................

...........................................................................................................................................

...........................................................................................................................................

.......................................................................................................................................[3]

© UCLES 2018 0610/43/O/N/18 [Turn over


8

(b) Stomata are found on the lower surface of broad bean leaves.

Describe the function of stomata.

...................................................................................................................................................

...................................................................................................................................................

...................................................................................................................................................

...................................................................................................................................................

...................................................................................................................................................

...................................................................................................................................................

...............................................................................................................................................[3]

(c) More than 40 years ago, botanists studied the leaves of broad bean plants and discovered
that guard cells control the opening and closing of stomata. They found that stomata were
open when the guard cells were turgid.

Table 3.1 shows some of their measurements.

Table 3.1

closed stomata open stomata


ion concentration in guard cells / pmol 0.3 2.5
guard cell volume / μm3 4000.0 6500.0
turgor pressure in the guard cells / MPa 2.0 4.8
width of stomatal opening / μm 0.0 8.0

(i) Ions move into guard cells by active transport.

Describe how the ions move into the guard cells.

...........................................................................................................................................

...........................................................................................................................................

.......................................................................................................................................[2]

© UCLES 2018 0610/43/O/N/18


9

(ii) Describe and explain how the change in ion concentration causes the guard cell volume
to change. Use the information in Table 3.1 in your answer.

...........................................................................................................................................

...........................................................................................................................................

...........................................................................................................................................

...........................................................................................................................................

...........................................................................................................................................

...........................................................................................................................................

...........................................................................................................................................

...........................................................................................................................................

...........................................................................................................................................

...........................................................................................................................................

.......................................................................................................................................[5]

(iii) The botanists left the broad bean plants unattended for three days. During this time the
broad bean plants wilted.

Suggest two environmental factors that can cause plants to wilt.

1 ........................................................................................................................................

2 ........................................................................................................................................
[2]

[Total: 18]

© UCLES 2018 0610/43/O/N/18 [Turn over


11

4 Glycogen is a storage carbohydrate in animals. Glycogen is made from glucose.

(a) (i) Cells that convert glucose to glycogen contain many mitochondria.

Suggest why these cells contain many mitochondria.

...........................................................................................................................................

...........................................................................................................................................

...........................................................................................................................................

.......................................................................................................................................[2]

(ii) State the type of biological molecule that catalyses reactions such as the conversion of
glycogen to glucose.

.......................................................................................................................................[1]

(b) A fetus needs glucose to make glycogen.

Describe how a fetus obtains glucose.

...................................................................................................................................................

...................................................................................................................................................

...................................................................................................................................................

...................................................................................................................................................

...................................................................................................................................................

...................................................................................................................................................

...............................................................................................................................................[3]

© UCLES 2018 0610/43/O/N/18 [Turn over


12

(c) Fig. 4. 1 shows the concentration of glycogen in the fetus of a domestic cat during pregnancy
and immediately after birth.

birth

700

600

500
glycogen
concentration 400
/ μmol per g
300

200

100

0
0 10 20 30 40 50 60 70 80 90
time since fertilisation / days

Fig. 4.1

Hormones stimulate changes in the concentration of glycogen in the fetus.

(i) Define the term hormone.

...........................................................................................................................................

...........................................................................................................................................

...........................................................................................................................................

...........................................................................................................................................

.......................................................................................................................................[3]

© UCLES 2018 0610/43/O/N/18


13

(ii) Calculate the percentage increase in the glycogen concentration in the fetus between
day 10 and birth in Fig. 4.1.

Give your answer to the nearest whole number.

Show your working.

............................................................ %
[2]

(iii) Describe the changes in glycogen concentration shown in Fig. 4.1 and explain how
hormones in the fetus cause these changes.

Use data from Fig. 4.1 to support your answer.

...........................................................................................................................................

...........................................................................................................................................

...........................................................................................................................................

...........................................................................................................................................

...........................................................................................................................................

...........................................................................................................................................

...........................................................................................................................................

...........................................................................................................................................

...........................................................................................................................................

...........................................................................................................................................

...........................................................................................................................................

...........................................................................................................................................

.......................................................................................................................................[6]

© UCLES 2018 0610/43/O/N/18 [Turn over


14

(d) After birth, cats produce milk to feed their offspring.

Human babies can be breast-fed or bottle-fed with formula milk.

Outline three disadvantages of breast-feeding.

1 ................................................................................................................................................

...................................................................................................................................................

2 ................................................................................................................................................

...................................................................................................................................................

3 ................................................................................................................................................

...................................................................................................................................................
[3]

[Total: 20]

© UCLES 2018 0610/43/O/N/18


15

5 An ecologist studied variation in a species of xerophyte.

(a) Xerophytes are adapted to a particular type of environment.

State this type of environment.

...............................................................................................................................................[1]

(b) The ecologist studied the features of the leaves in the species of xerophyte.

Fig. 5.1 shows the variation in the type of leaf spike.

number of 5
individuals
4

0
type 1 type 2 type 3
type of leaf spike

Fig. 5.1

(i) State the type of variation shown in Fig. 5.1.

.......................................................................................................................................[1]

(ii) Explain why the type of leaf spike is an example of the variation shown in Fig. 5.1.

...........................................................................................................................................

...........................................................................................................................................

.......................................................................................................................................[2]

© UCLES 2018 0610/43/O/N/18 [Turn over


16

(c) The ecologist also measured other features of the leaves.

Fig. 5.2 shows the variation in leaf feature B.

number of 5
individuals
4

0
0–5 6–10 11–15 16–20 21–25
leaf feature B / arbitrary units

Fig. 5.2

State two named features of leaves that show the type of variation shown in Fig. 5.2.

1 ................................................................................................................................................

2 ................................................................................................................................................
[2]

© UCLES 2018 0610/43/O/N/18


17

(d) After one year, the ecologist recorded the variation in leaf feature B again.

The results are shown in Fig. 5.3.

number of 5
individuals
4

0
0–5 6–10 11–15 16–20 21–25
leaf feature B / arbitrary units

Fig. 5.3

Suggest one reason for the difference in variation of leaf feature B after one year.

...................................................................................................................................................

...............................................................................................................................................[1]

[Total: 7]

© UCLES 2018 0610/43/O/N/18 [Turn over


18

6 Fig. 6.1 is a diagram showing some body cells and parts of the human lymphatic and circulatory
systems.

arteriole

lymphatic vessel
B

Z capillary

venule

not to scale

Fig. 6.1

(a) Capillaries allow blood to reach most cells in the body.

(i) State the name of the process by which oxygen moves from A to Z as shown in Fig. 6.1.

.......................................................................................................................................[1]

(ii) Describe how some of the liquid in A moves to B in Fig. 6.1.

...........................................................................................................................................

...........................................................................................................................................

.......................................................................................................................................[2]

(iii) State one component of blood that remains inside the capillaries as the blood flows from
X to Y in Fig. 6.1.

.......................................................................................................................................[1]

© UCLES 2018 0610/43/O/N/18


19

(b) Lymphatic vessels are similar in structure to veins.

(i) Describe the structure of veins.

...........................................................................................................................................

...........................................................................................................................................

...........................................................................................................................................

...........................................................................................................................................

.......................................................................................................................................[2]

(ii) Describe the role of the lymphatic vessel shown in Fig. 6.1.

...........................................................................................................................................

...........................................................................................................................................

...........................................................................................................................................

...........................................................................................................................................

.......................................................................................................................................[2]

(c) Lacteals are another part of the lymphatic system.

State where in the body lacteals are found and state their function.

location in the body ...................................................................................................................

function .....................................................................................................................................

...................................................................................................................................................
[2]

(d) In the lymphatic system, there are structures that contain large numbers of lymphocytes.

(i) State the name of these structures.

.......................................................................................................................................[1]

(ii) State the role of lymphocytes.

...........................................................................................................................................

...........................................................................................................................................

.......................................................................................................................................[2]

[Total: 13]

© UCLES 2018 0610/43/O/N/18


Cambridge Assessment International Education
Cambridge International General Certificate of Secondary Education
* 9 8 4 9 3 1 5 0 1 1 *

BIOLOGY 0610/43
Paper 4 Theory (Extended) October/November 2019
1 hour 15 minutes
Candidates answer on the Question Paper.
No Additional Materials are required.

READ THESE INSTRUCTIONS FIRST

Write your centre number, candidate number and name on all the work you hand in.
Write in dark blue or black pen.
You may use an HB pencil for any diagrams or graphs.
Do not use staples, paper clips, glue or correction fluid.
DO NOT WRITE IN ANY BARCODES.

Answer all questions.

Electronic calculators may be used.


You may lose marks if you do not show your working or if you do not use appropriate units.

At the end of the examination, fasten all your work securely together.
The number of marks is given in brackets [ ] at the end of each question or part question.

This syllabus is regulated for use in England, Wales and Northern Ireland as a Cambridge International Level 1/Level 2 Certificate.

This document consists of 16 printed pages and 4 blank pages.

DC (ST/CT) 173044/4
© UCLES 2019 [Turn over
2

1 (a) The ant-mimic jumping spider, Myrmarachne formicaria, is shown in Fig. 1.1.

The common name of this species describes its behaviour. It is an arachnid that tricks its prey
because it looks like the insects that it eats.

Fig. 1.1

(i) Suggest which trophic level in a food chain M. formicaria could belong to.

..................................................................................................................................... [1]

(ii) State the genus of the spider shown in Fig. 1.1.

..................................................................................................................................... [1]

(iii) Some keys use paired choices of features to identify species such as the ant-mimic
jumping spider.

State the name of this type of key.

..................................................................................................................................... [1]

© UCLES 2019 0610/43/O/N/19


3

(b) Spiders are classified as arachnids. Arachnids are one of the main groups of arthropods.

Fig. 1.2 shows diagrams of six arthropods, four of which are arachnids.

A B C

D E F not to scale

Fig. 1.2

(i) State two common features of all the arthropods, visible in Fig. 1.2.

1 ........................................................................................................................................

2 ........................................................................................................................................
[2]

(ii) State two common features of all arachnids that can be used to distinguish them from
other arthropods.

1 ........................................................................................................................................

2 ........................................................................................................................................
[2]

(iii) State the letters of the four arachnids shown in Fig. 1.2.

..................................................................................................................................... [2]

(c) The features shown in Fig. 1.2 are morphological features. Many traditional methods of
classification used morphology.

State the name of one other type of feature that can also be used in classification.

............................................................................................................................................. [1]

[Total: 10]
© UCLES 2019 0610/43/O/N/19 [Turn over
4

2 Plants produce glucose in leaves and convert some of it to sucrose.

(a) (i) Explain how glucose is produced in leaves.

...........................................................................................................................................

...........................................................................................................................................

...........................................................................................................................................

...........................................................................................................................................

...........................................................................................................................................

...........................................................................................................................................

..................................................................................................................................... [3]

(ii) State the name of the process that plants use to move sucrose from a source to a sink.

..................................................................................................................................... [1]

(iii) Roots can be an example of a sink.

Explain why sometimes roots act as a source rather than a sink.

...........................................................................................................................................

...........................................................................................................................................

...........................................................................................................................................

...........................................................................................................................................

..................................................................................................................................... [2]

© UCLES 2019 0610/43/O/N/19


6

(b) The movement of sucrose in plants can be modelled using laboratory apparatus.

Fig. 2.1 shows the apparatus used to model the movement of sucrose in a plant:

• Partially permeable bags were attached tightly to the ends of tube Q.


• The bag representing a source was filled with a coloured sucrose solution.
• The bag representing a sink was filled with water.
• The containers and tube Q and tube S were filled with water.

tube Q

bag representing a
source containing a
coloured sucrose
solution bag representing
a sink containing
partially water
permeable
membrane P partially permeable
membrane R
water
tube S

Fig. 2.1

Fig. 2.2 shows the position of the coloured sucrose solution 30 minutes after the apparatus
was set up.

The arrows show the direction of the movement of the liquids.

movement of sucrose solution


tube Q

bag representing a
source
bag representing
a sink
partially
permeable
membrane P partially permeable
membrane R
water
tube S

movement of water

Fig. 2.2

© UCLES 2019 0610/43/O/N/19


7

(i) State the name of the tissue represented by tube Q and the name of the tissue
represented by tube S in Fig. 2.2.

Q .......................................................................................................................................

S ........................................................................................................................................
[2]

(ii) Explain why the sucrose solution moves along tube Q in the model in Fig. 2.2.

...........................................................................................................................................

...........................................................................................................................................

...........................................................................................................................................

...........................................................................................................................................

...........................................................................................................................................

...........................................................................................................................................

...........................................................................................................................................

...........................................................................................................................................

..................................................................................................................................... [4]

(c) In plants the movement of sucrose is usually continuous. However, after 2 hours the
movement of sucrose in tube Q in Fig. 2.2 stopped.

Suggest why the movement of sucrose in tube Q stopped.

...................................................................................................................................................

...................................................................................................................................................

............................................................................................................................................. [1]

(d) Amino acids are also transported through plants.

State the name of the mineral ion that is used to make amino acids.

............................................................................................................................................. [1]

[Total: 14]

© UCLES 2019 0610/43/O/N/19 [Turn over


8

3 Carbohydrates are an important component of a balanced diet.

The flow chart in Fig. 3.1 shows some of the processes that happen to carbohydrates in food that
is eaten.

ingestion of large biological molecules

chemical digestion

absorption into the blood via the alimentary canal

increase in blood glucose concentration

assimilation in the liver

Fig. 3.1

© UCLES 2019 0610/43/O/N/19


9

(a) Describe what happens to carbohydrates in the human body between ingestion and
assimilation in the liver.

Use the information in Fig. 3.1 as a guide.

...................................................................................................................................................

...................................................................................................................................................

...................................................................................................................................................

...................................................................................................................................................

...................................................................................................................................................

...................................................................................................................................................

...................................................................................................................................................

...................................................................................................................................................

...................................................................................................................................................

...................................................................................................................................................

...................................................................................................................................................

...................................................................................................................................................

...................................................................................................................................................

...................................................................................................................................................

...................................................................................................................................................

...................................................................................................................................................

...................................................................................................................................................

...................................................................................................................................................

...................................................................................................................................................

............................................................................................................................................. [8]

© UCLES 2019 0610/43/O/N/19 [Turn over


11

(b) Mineral salts are another important component of a balanced diet.

State the importance of calcium ions and iron ions in a balanced diet.

calcium ions ..............................................................................................................................

...................................................................................................................................................

...................................................................................................................................................

iron ions ....................................................................................................................................

...................................................................................................................................................

...................................................................................................................................................
[4]

(c) Consuming too much of some mineral salts, such as sodium chloride, increases the risk of
developing coronary heart disease (CHD).

Doctors studied the effect of diet on the risk of developing CHD.

The doctors first selected volunteers who had a high salt diet.

The doctors assessed the volunteers’ overall risk of developing CHD and monitored their
blood pressure.

(i) List two factors, other than diet and blood pressure, that the doctors considered when
assessing the overall risk of the volunteers developing CHD.

1 ........................................................................................................................................

2 ........................................................................................................................................
[2]

(ii) The doctors used urine tests to identify volunteers who had a high salt diet.

Explain why urine tests are a good indicator of how much salt has been consumed.

...........................................................................................................................................

...........................................................................................................................................

...........................................................................................................................................

...........................................................................................................................................

..................................................................................................................................... [2]

© UCLES 2019 0610/43/O/N/19 [Turn over


12

(d) The volunteers were divided into two groups.

The mass of salt consumed by both groups was changed every 4 weeks:
• low salt intake for 4 weeks
• medium salt intake for 4 weeks
• high salt intake for 4 weeks.

In addition, group 2 was given other changes to their diet but group 1 was not.

(i) Suggest one component of the diet of group 2, other than salt, that the doctors changed
to further reduce the risk of developing CHD.

..................................................................................................................................... [1]

The systolic blood pressure of the volunteers was measured every 4 weeks.
These results are shown in Fig. 3.2.

18.0
Key:
17.5 group 1
group 2
average systolic 17.0
blood pressure
/ kPa 16.5

16.0

15.5

15.0
low medium high
salt intake

Fig. 3.2

(ii) Calculate the percentage increase in the average systolic blood pressure of the group 1
volunteers when they increased their salt intake from low to high.

low salt intake ............................... kPa

high salt intake ............................... kPa

Give your answer to the nearest whole number.

Space for working.

.......................................................... %
[3]

© UCLES 2019 0610/43/O/N/19


13

(iii) The doctors concluded that some diets reduce the risk of CHD.

Give evidence from Fig. 3.2 to support this conclusion.

...........................................................................................................................................

...........................................................................................................................................

...........................................................................................................................................

...........................................................................................................................................

...........................................................................................................................................

...........................................................................................................................................

..................................................................................................................................... [3]

[Total: 23]

4 Forest ecosystems are threatened by many human activities.

(a) (i) Describe reasons why people cut down forests.

...........................................................................................................................................

...........................................................................................................................................

...........................................................................................................................................

...........................................................................................................................................

...........................................................................................................................................

...........................................................................................................................................

..................................................................................................................................... [3]

(ii) Describe how forests can be conserved.

...........................................................................................................................................

...........................................................................................................................................

...........................................................................................................................................

...........................................................................................................................................

...........................................................................................................................................

...........................................................................................................................................

..................................................................................................................................... [3]

© UCLES 2019 0610/43/O/N/19 [Turn over


14

(b) Ecologists in one country were concerned that some mammals had been affected by
deforestation.

Fig. 4.1 is a diagram showing how deforestation affected one area of forest.

Key:
forest
cleared land

original forest

many trees were cut down

small forest area

remaining forest

large forest area

Fig. 4.1

The ecologists studied eight species of mammal. They recorded how many of the small and
large areas of the remaining forest contained each of the eight species.

Two species, the black rat and the house mouse, are not usually found in this forest
ecosystem. They were introduced to the area by humans many years ago before any trees
were cut down.

The other six species are known to live in this forest ecosystem.

© UCLES 2019 0610/43/O/N/19


15

Table 4.1

percentage of small percentage of large


average body
species of mammal forest areas containing forest areas containing
mass / g
the species the species
house mouse* 18 46 42
brown antechinus 40 62 83
swamp rat 130 15 25
bush rat 160 85 100
black rat* 200 15 0
southern brown bandicoot 850 31 92
long-nosed potoroo 1100 8 17
long-nosed bandicoot 1300 8 25
*introduced species

(i) State which mammal in Table 4.1 showed almost no preference between small and large
areas of forest.

..................................................................................................................................... [1]

(ii) The ecologists made a hypothesis:

‘Larger areas of forest are better for the conservation of mammals.’

Discuss the evidence from Table 4.1 to support or reject this hypothesis.

...........................................................................................................................................

...........................................................................................................................................

...........................................................................................................................................

...........................................................................................................................................

...........................................................................................................................................

...........................................................................................................................................

...........................................................................................................................................

...........................................................................................................................................

..................................................................................................................................... [4]

[Total: 11]

© UCLES 2019 0610/43/O/N/19 [Turn over


16

5 Genetic engineering is an example of an important biotechnology.

(a) Complete the passage below by filling in each space with a suitable word.

DNA is a biological molecule found in the ................................................. of an animal cell.

The sequence of the ................................................. in mRNA determines the order of the

amino acids that will be assembled into a ................................................. .

When carrying out genetic engineering, sections of human DNA called

................................................. are cut using restriction enzymes. Next bacterial plasmids

are cut with the ................................................. restriction enzymes to form complementary

................................................. ends.

The cut section of human DNA is inserted into the cut plasmid and they are joined together

to form a ................................................. plasmid. These plasmids are inserted into

................................................. and replication occurs. This process is used to produce

human ................................................. that is used to treat people with diabetes.


[9]

(b) In addition to genetic engineering, enzymes are also useful in other biotechnologies.

State two examples of how enzymes are used in another biotechnology.

1 ................................................................................................................................................

2 ................................................................................................................................................
[2]

[Total: 11]

© UCLES 2019 0610/43/O/N/19


17

6 Fig. 6.1 is a photomicrograph of a blood clot.

L
J

Fig. 6.1

(a) Describe how a blood clot forms.

Use the letters in Fig. 6.1 in your answer.

...................................................................................................................................................

...................................................................................................................................................

...................................................................................................................................................

...................................................................................................................................................

...................................................................................................................................................

...................................................................................................................................................

...................................................................................................................................................

...................................................................................................................................................

...................................................................................................................................................

...................................................................................................................................................

............................................................................................................................................. [5]

© UCLES 2019 0610/43/O/N/19 [Turn over


18

(b) State the importance of blood clotting.

...................................................................................................................................................

...................................................................................................................................................

...................................................................................................................................................

...................................................................................................................................................

............................................................................................................................................. [2]

(c) There are four blood group phenotypes A, B, AB and O in humans.

(i) Define the term phenotype.

...........................................................................................................................................

...........................................................................................................................................

..................................................................................................................................... [1]

(ii) State the name of the type of inheritance that is shown by blood groups.

..................................................................................................................................... [1]

(iii) State the two possible genotypes for a person who has the phenotype blood group A.

1 ........................................................................................................................................

2 ........................................................................................................................................
[2]

[Total: 11]

© UCLES 2019 0610/43/O/N/19


Cambridge IGCSE™
* 1 4 2 8 9 9 0 7 7 1 *

BIOLOGY 0610/43
Paper 4 Theory (Extended) October/November 2020

1 hour 15 minutes

You must answer on the question paper.

No additional materials are needed.

INSTRUCTIONS
● Answer all questions.
● Use a black or dark blue pen. You may use an HB pencil for any diagrams or graphs.
● Write your name, centre number and candidate number in the boxes at the top of the page.
● Write your answer to each question in the space provided.
● Do not use an erasable pen or correction fluid.
● Do not write on any bar codes.
● You may use a calculator.
● You should show all your working and use appropriate units.

INFORMATION
● The total mark for this paper is 80.
● The number of marks for each question or part question is shown in brackets [ ].

This document has 16 pages. Blank pages are indicated.

DC (JC/CT) 188259/4
© UCLES 2020 [Turn over
2

1 Plants are sensitive to light.

(a) Fig. 1.1 shows a sequence of diagrams for two young plant seedlings. The seedlings were
first grown in pots with light from all directions and then placed into boxes.

• Seedling A was placed in a box with light entering from one side only.
• Seedling B was placed in a box with no light.

seedling

soil
water
A A

B B

Fig. 1.1

Complete Fig. 1.1 by drawing seedling A and seedling B, in the boxes, to show the expected
direction of their growth after one week. [2]

© UCLES 2020 0610/43/O/N/20


3

(b) Plants are also sensitive to gravity.

(i) Define the term sensitivity.

...........................................................................................................................................

...........................................................................................................................................

..................................................................................................................................... [2]

(ii) Explain the role of plant hormones in gravitropism.

...........................................................................................................................................

...........................................................................................................................................

...........................................................................................................................................

...........................................................................................................................................

...........................................................................................................................................

...........................................................................................................................................

...........................................................................................................................................

...........................................................................................................................................

..................................................................................................................................... [4]

(iii) Gravitropism enables plants to survive in their environment.

Explain how gravitropism enables a plant to survive.

...........................................................................................................................................

...........................................................................................................................................

...........................................................................................................................................

...........................................................................................................................................

..................................................................................................................................... [2]

© UCLES 2020 0610/43/O/N/20 [Turn over


4

(c) Reflexes in animals are also an example of sensitivity.

(i) Describe how reflexes in animals differ from sensitivity in plants.

...........................................................................................................................................

...........................................................................................................................................

...........................................................................................................................................

...........................................................................................................................................

..................................................................................................................................... [2]

(ii) The pupil reflex is an example of a reflex that occurs in the eye. The eye is a sense
organ.

Define the term sense organ.

...........................................................................................................................................

...........................................................................................................................................

...........................................................................................................................................

...........................................................................................................................................

..................................................................................................................................... [2]

[Total: 14]

© UCLES 2020 0610/43/O/N/20


5

2 (a) A person ate a meal containing starch and fat.

Fig. 2.1 shows some events that occurred after ingesting this meal.

P absorption of nutrients in the villi

Q assimilation of fatty acids in the liver

R breakdown of large food particles by the teeth

S movement of small food particles through the oesophagus

T secretion of amylase from the salivary glands

Fig. 2.1

Put the events in Fig. 2.1 into the correct sequence. One has been done for you.

R
[2]

(b) The stomach lining contains cells that secrete proteins.

(i) State the names of two cell structures that are involved in making or secreting proteins.

1 ........................................................................................................................................

2 ........................................................................................................................................
[2]

(ii) State the name of one component of gastric juice and describe its functions.

...........................................................................................................................................

...........................................................................................................................................

...........................................................................................................................................

...........................................................................................................................................

...........................................................................................................................................

...........................................................................................................................................

...........................................................................................................................................
[3]

© UCLES 2020 0610/43/O/N/20 [Turn over


6

(iii) There are goblet cells in many parts of the alimentary canal.

State the substance that goblet cells secrete.

..................................................................................................................................... [1]

(c) Emulsification of fats occurs in the alimentary canal.

(i) Describe the process of emulsification of fats.

...........................................................................................................................................

...........................................................................................................................................

...........................................................................................................................................

...........................................................................................................................................

..................................................................................................................................... [2]

(ii) State the name of the organ in the alimentary canal where fats are emulsified.

..................................................................................................................................... [1]

(iii) State the name of the structure in the villi where digested fats are absorbed.

..................................................................................................................................... [1]

[Total: 12]

3 (a) Fig. 3.1 is part of a food web in a rainforest.

anaconda
jaguar

iguana frog

monkey grasshopper fruit bat

coconut tree banana plant

Fig. 3.1

© UCLES 2020 0610/43/O/N/20


7

(i) Complete Fig. 3.2 using the information in Fig. 3.1.

The first one has been done for you.

the number of different producers 2

the number of different secondary consumers

the number of different trophic levels in the food web

how many different trophic levels monkeys feed at

Fig. 3.2
[3]

(ii) Sometimes anacondas are able to kill and eat jaguars.

Using the information in Fig. 3.1, state the name of the highest possible trophic level of
an anaconda that eats a jaguar.

..................................................................................................................................... [1]

(iii) Explain why the population of anacondas is always smaller than the population of fruit
bats in the food web shown in Fig. 3.1.

Use the concept of energy flow in your answer.

...........................................................................................................................................

...........................................................................................................................................

...........................................................................................................................................

...........................................................................................................................................

...........................................................................................................................................

...........................................................................................................................................

..................................................................................................................................... [3]

© UCLES 2020 0610/43/O/N/20 [Turn over


8

(b) (i) A giant rat was discovered in a natural rainforest on Vangunu Island. Scientists wanted
to determine if it was a new species.

Explain how scientists can use tissue samples to classify this rat.

...........................................................................................................................................

...........................................................................................................................................

...........................................................................................................................................

...........................................................................................................................................

..................................................................................................................................... [2]

(ii) Giant rats eat coconuts.

Coconuts are grown as a monoculture for human consumption.

Suggest how monocultures of crop plants can result in the extinction of some animals.

...........................................................................................................................................

...........................................................................................................................................

...........................................................................................................................................

...........................................................................................................................................

...........................................................................................................................................

...........................................................................................................................................

..................................................................................................................................... [3]

[Total: 12]

© UCLES 2020 0610/43/O/N/20


9

4 Yeast is an example of a single-celled fungus.

(a) The population of yeast can be estimated by using a light microscope to view a small sample
of yeast cells in a modified microscope slide.

There is a square chamber, with a specific volume, cut into the microscope slide and a grid to
make it easier to count the number of cells.

Fig. 4.1 shows a diagram of the modified microscope slide. It also shows part of a light
microscope with the modified microscope slide filled to the top with a sample of yeast cells.

grid

chamber

Fig. 4.1

The length of the sides of each small square in the grid in the modified microscope slide is
200 μm. The depth of the chamber is 100 μm. There is a total of 25 small squares in the grid.

Each small square in the grid has an average of 52 yeast cells.

Calculate the concentration of yeast cells per mm3.

Space for working.

......................................... cells per mm3


[3]

© UCLES 2020 0610/43/O/N/20 [Turn over


10

(b) A small sample of yeast cells and 19 g per dm3 of sugar were added to a fermenter. The
population size of live yeast cells and the concentration of the sugar in the fermenter were
calculated every day for 10 days.

The results are shown in Fig. 4.2 and Fig. 4.3.

20

20
18

18
16

16
14

concentration 12
14
of sugar
/concentration
g per dm3 12
10
of sugar
/ g per dm3 10
8

8
6

6
4

4
2

2
0
0 1 2 3 4 5 6 7 8 9 10
0 time / days
0 1 2 3 4 5 6 7 8 9 10
Fig. 4.2time / days

population size
of live yeast cells
population size
of live yeast cells

0 1 2 3 4 5 6 7 8 9 10
time / days
0 1 2 3 4 5 6 7 8 9 10
time / days

Fig. 4.3
© UCLES 2020 0610/43/O/N/20
11

(i) Draw an X on Fig. 4.3 to indicate the position of the lag phase. [1]

(ii) Complete the line on Fig. 4.3 to show the expected change in the population size of live
yeast cells from day 3 to day 10. [2]

(iii) Describe and explain the changes in the concentration of sugar in the fermenter over the
10-day period.

...........................................................................................................................................

...........................................................................................................................................

...........................................................................................................................................

...........................................................................................................................................

...........................................................................................................................................

...........................................................................................................................................

...........................................................................................................................................

...........................................................................................................................................

...........................................................................................................................................

...........................................................................................................................................

..................................................................................................................................... [5]

(c) Explain why it is important that there is a tube in the fermenter that allows gases to escape.

...................................................................................................................................................

...................................................................................................................................................

...................................................................................................................................................

...................................................................................................................................................

............................................................................................................................................. [2]

[Total: 13]

© UCLES 2020 0610/43/O/N/20 [Turn over


12

5 Fig. 5.1 shows a photomicrograph of a sperm cell reaching an egg cell.

Fig. 5.1

(a) (i) Complete the sentence:

A nucleus containing a single set of unpaired chromosomes in a sperm is called a

............................................... nucleus. [1]

(ii) State where, in the female reproductive system, the event shown in Fig. 5.1 occurs.

..................................................................................................................................... [1]

(iii) Describe what happens from the event shown in Fig. 5.1 until an embryo is formed.

...........................................................................................................................................

...........................................................................................................................................

...........................................................................................................................................

...........................................................................................................................................

...........................................................................................................................................

...........................................................................................................................................

...........................................................................................................................................

...........................................................................................................................................

..................................................................................................................................... [4]

© UCLES 2020 0610/43/O/N/20


13

(b) Some people use fertility drugs and in vitro fertilisation (IVF) to have children.

Outline the use of fertility drugs and the process of IVF.

...................................................................................................................................................

...................................................................................................................................................

...................................................................................................................................................

...................................................................................................................................................

...................................................................................................................................................

...................................................................................................................................................

...................................................................................................................................................

...................................................................................................................................................

...................................................................................................................................................

...................................................................................................................................................

...................................................................................................................................................

...................................................................................................................................................

............................................................................................................................................. [6]

[Total: 12]

© UCLES 2020 0610/43/O/N/20 [Turn over


14

6 (a) Researchers investigated four different insecticides to determine how effective they were at
removing three different species of insects, A, B and C, from crop plants.

They tested different concentrations of each insecticide to find the minimum dosage required
to remove 95% of each insect population in two hours.

Each insecticide had the same cost per gram.

The results are shown in Table 6.1.

Table 6.1

minimum dosage required


species of / mg per dm3
insect
insecticide 1 insecticide 2 insecticide 3 insecticide 4
A 3 51 5 58
B 31 27 2 75
C 10 2 3 65

(i) Use the information in Table 6.1 to decide which one of the four different insecticides and
which dosage would be the best choice for removing insects A, B and C from a field of
crop plants.

Explain your choices.

insecticide ............................................... dosage .............................................................

explanation ........................................................................................................................

...........................................................................................................................................

...........................................................................................................................................

...........................................................................................................................................

...........................................................................................................................................

...........................................................................................................................................
[3]

© UCLES 2020 0610/43/O/N/20


15

(ii) Discuss factors, other than dosage, that should be considered to prevent the insecticide
damaging the environment.

...........................................................................................................................................

...........................................................................................................................................

...........................................................................................................................................

...........................................................................................................................................

...........................................................................................................................................

...........................................................................................................................................

..................................................................................................................................... [3]

(b) (i) Describe the advantages to farmers of using insecticides.

...........................................................................................................................................

...........................................................................................................................................

...........................................................................................................................................

...........................................................................................................................................

..................................................................................................................................... [2]

(ii) Farmers also use other chemicals on their crop plants.

State the names of chemicals, other than insecticides, that are used by farmers on crop
plants and explain their benefits.

...........................................................................................................................................

...........................................................................................................................................

...........................................................................................................................................

...........................................................................................................................................

...........................................................................................................................................

...........................................................................................................................................

..................................................................................................................................... [3]

© UCLES 2020 0610/43/O/N/20 [Turn over


16

(c) Insecticides are often made from chemicals that occur naturally in plants. Tobacco plants are
one example of this.

(i) Suggest why having a natural insecticide is an adaptive feature of plants.

...........................................................................................................................................

...........................................................................................................................................

..................................................................................................................................... [1]

(ii) Describe why people find it difficult to stop smoking tobacco.

...........................................................................................................................................

...........................................................................................................................................

...........................................................................................................................................

...........................................................................................................................................

..................................................................................................................................... [2]

(iii) State the names of two diseases that can be caused by smoking tobacco.

1 ........................................................................................................................................

2 ........................................................................................................................................
[2]

(iv) State the name of the component of tobacco that can damage alveoli.

..................................................................................................................................... [1]

[Total: 17]

Permission to reproduce items where third-party owned material protected by copyright is included has been sought and cleared where possible. Every
reasonable effort has been made by the publisher (UCLES) to trace copyright holders, but if any items requiring clearance have unwittingly been included, the
publisher will be pleased to make amends at the earliest possible opportunity.

To avoid the issue of disclosure of answer-related information to candidates, all copyright acknowledgements are reproduced online in the Cambridge
Assessment International Education Copyright Acknowledgements Booklet. This is produced for each series of examinations and is freely available to download
at www.cambridgeinternational.org after the live examination series.

Cambridge Assessment International Education is part of the Cambridge Assessment Group. Cambridge Assessment is the brand name of the University of
Cambridge Local Examinations Syndicate (UCLES), which itself is a department of the University of Cambridge.

© UCLES 2020 0610/43/O/N/20


Cambridge IGCSE™
* 3 9 5 6 9 4 5 6 1 7 *

BIOLOGY 0610/43
Paper 4 Theory (Extended) October/November 2021

1 hour 15 minutes

You must answer on the question paper.

No additional materials are needed.

INSTRUCTIONS
● Answer all questions.
● Use a black or dark blue pen. You may use an HB pencil for any diagrams or graphs.
● Write your name, centre number and candidate number in the boxes at the top of the page.
● Write your answer to each question in the space provided.
● Do not use an erasable pen or correction fluid.
● Do not write on any bar codes.
● You may use a calculator.
● You should show all your working and use appropriate units.

INFORMATION
● The total mark for this paper is 80.
● The number of marks for each question or part question is shown in brackets [ ].

This document has 20 pages. Any blank pages are indicated.

DC (LK) 308227/5 R
© UCLES 2021 [Turn over
2

1 Enzymes are used in genetic engineering.

(a) Define the term enzyme.

...................................................................................................................................................

...................................................................................................................................................

............................................................................................................................................. [2]

(b) The process of genetic engineering often starts with the steps shown in Fig. 1.1.

upper DNA strand

G A A T T C
step 1 lower DNA strand

step 2 enzyme 1

A A T T C
G
step 3

Fig. 1.1

(i) State the sequence of bases on the lower strand of the DNA molecule in step 1.

upper DNA strand G A A T T C

lower DNA strand

[1]

(ii) State the name of enzyme 1 in step 2 of Fig. 1.1.

..................................................................................................................................... [1]

© UCLES 2021 0610/43/O/N/21


3

(iii) Describe the effect of enzyme 1 on the DNA molecule in step 3.

...........................................................................................................................................

...........................................................................................................................................

...........................................................................................................................................

...........................................................................................................................................

..................................................................................................................................... [2]

(iv) Explain how enzyme 1 in Fig. 1.1 is specific to the exact sequence of DNA bases.

...........................................................................................................................................

...........................................................................................................................................

...........................................................................................................................................

...........................................................................................................................................

..................................................................................................................................... [2]

© UCLES 2021 0610/43/O/N/21 [Turn over


4

(c) Another enzyme, enzyme 2, is used later in the process of genetic engineering.

Fig. 1.2 is a diagram showing the action of enzyme 2.

plasmid DNA
strands

upper DNA strand


of the gene
lower DNA strand
step 6 of the gene

step 7 enzyme 2

step 8

Fig. 1.2

(i) Some organisms naturally contain DNA in the form of a plasmid.

State the name of the type of organism that naturally contains plasmids.

..................................................................................................................................... [1]

(ii) State the name of enzyme 2 in step 7 of Fig. 1.2.

..................................................................................................................................... [1]

(iii) State the name of the molecule formed in step 8.

..................................................................................................................................... [1]

© UCLES 2021 0610/43/O/N/21


5

(d) Sketch a graph to describe how the activity of the enzymes used in genetic engineering would
change if the reaction occurred at a range of temperatures from very cold to very hot.

Label the axes with appropriate titles.

Do not use units or a numbered scale.

[3]

[Total: 14]

© UCLES 2021 0610/43/O/N/21 [Turn over


6

2 Fig. 2.1 is a photomicrograph of the end of a plant root.

length / mm

root tip 0 5 10 15 20 25 30 35

Fig. 2.1

Fig. 2.2 shows the results of a study on the rate of uptake of nitrate ions at different points along
the root shown in Fig. 2.1.

30

25

20

rate of nitrate ion


uptake 15
2
/ pmol per cm per s

10

0
0 5 10 15 20 25 30 35
distance from root tip / mm

Fig. 2.2

© UCLES 2021 0610/43/O/N/21


7

(a) (i) Describe the rate of uptake of nitrate ions along the root.

Use the information in Fig. 2.1 and Fig. 2.2 in your answer.

...........................................................................................................................................

...........................................................................................................................................

...........................................................................................................................................

...........................................................................................................................................

...........................................................................................................................................

...........................................................................................................................................

...........................................................................................................................................

...........................................................................................................................................

...........................................................................................................................................

...........................................................................................................................................

..................................................................................................................................... [5]

(ii) Explain how nitrate ions move from the soil into roots.

...........................................................................................................................................

...........................................................................................................................................

...........................................................................................................................................

...........................................................................................................................................

...........................................................................................................................................

...........................................................................................................................................

...........................................................................................................................................

...........................................................................................................................................

..................................................................................................................................... [4]

(iii) Explain why the uptake of ions, such as nitrate, is important for the uptake of water in
roots.

...........................................................................................................................................

...........................................................................................................................................

..................................................................................................................................... [1]

© UCLES 2021 0610/43/O/N/21 [Turn over


8

(iv) Explain why plants need nitrate ions, other than for the uptake of water.

...........................................................................................................................................

...........................................................................................................................................

...........................................................................................................................................

...........................................................................................................................................

...........................................................................................................................................

...........................................................................................................................................

..................................................................................................................................... [3]

(v) Describe how nitrate ions are formed in the soil.

...........................................................................................................................................

...........................................................................................................................................

...........................................................................................................................................

...........................................................................................................................................

...........................................................................................................................................

...........................................................................................................................................

..................................................................................................................................... [3]

© UCLES 2021 0610/43/O/N/21


9

(b) (i) Fig. 2.3 shows some of the events that occur when high concentrations of nitrate ions
flow into lakes.

A a decrease in the concentration of dissolved oxygen

B a decrease in the population of consumers

C a decrease in the population of producers

D a decrease in light intensity at the bottom of the lake

E an increase in the population of decomposers

F an increase in the population of producers

Fig. 2.3

Put the events shown in Fig. 2.3 into the correct sequence.

[2]

(ii) State the name of the process summarised in Fig. 2.3.

..................................................................................................................................... [1]

[Total: 19]

© UCLES 2021 0610/43/O/N/21 [Turn over


10

3 A researcher investigated genetic variation in fruit flies, Drosophila melanogaster.

The bodies of fruit flies can be black or yellow. A yellow body colour is a recessive feature in fruit
flies.

(a) Two heterozygous fruit flies with black bodies were bred together.

Predict the phenotypes of the offspring and the phenotypic ratio for this cross.

............................................................................................................................................. [1]

(b) In another fruit fly breeding experiment, researchers counted 124 offspring with black bodies
and 121 offspring with yellow bodies.

Draw a genetic diagram to explain the results of this cross. Use the letter B to represent the
allele for black body colour and the letter b to represent the allele for yellow body colour.

parental phenotypes .................................. × ..................................

parental genotypes .................................. × ..................................

gametes , × ,
............. ............. ............. .............

offspring genotypes ..................................................................................................................

expected offspring phenotype ratio ...................... black : ...................... yellow

actual offspring phenotype ratio 124 black : 121 yellow


[5]

© UCLES 2021 0610/43/O/N/21


11

(c) The crab, Cerberusa caeca, lives in dark caves and has no coloured pigment.

Fig. 3.1 is a photograph of C. caeca.

Fig. 3.1

(i) C. caeca and D. melanogaster are both arthropods.

State one feature present in all arthropods but not present in vertebrates.

..................................................................................................................................... [1]

(ii) C. caeca is a crustacean and D. melanogaster is an insect.

State one morphological feature of C. caeca that distinguishes it as a crustacean and


not as an insect.

..................................................................................................................................... [1]

© UCLES 2021 0610/43/O/N/21 [Turn over


12

(d) The ancestors of C. caeca had pigmented bodies.

The lack of a coloured pigment in C. caeca is called albinism and was caused by a mutation
many thousands of years ago.

(i) Explain the mechanism that has resulted in the allele for albinism becoming common in
recent generations in populations of C. caeca.

...........................................................................................................................................

...........................................................................................................................................

...........................................................................................................................................

...........................................................................................................................................

...........................................................................................................................................

...........................................................................................................................................

..................................................................................................................................... [3]

(ii) State two factors that can cause mutations.

1 ........................................................................................................................................

2 ........................................................................................................................................
[2]

[Total: 13]

© UCLES 2021 0610/43/O/N/21


13

4 HIV is a pathogen that can cause AIDS.

(a) Describe how HIV is transmitted from one person to another.

...................................................................................................................................................

...................................................................................................................................................

...................................................................................................................................................

...................................................................................................................................................

...................................................................................................................................................

...................................................................................................................................................

............................................................................................................................................. [3]

(b) All viruses contain genetic material. HIV contains genetic material called RNA.

State one other feature common to all viruses.

............................................................................................................................................. [1]

(c) (i) Describe the function of lymphocytes.

...........................................................................................................................................

...........................................................................................................................................

...........................................................................................................................................

...........................................................................................................................................

...........................................................................................................................................

...........................................................................................................................................

..................................................................................................................................... [3]

(ii) State how infection with HIV affects the lymphocytes if untreated.

...........................................................................................................................................

..................................................................................................................................... [1]

© UCLES 2021 0610/43/O/N/21 [Turn over


14

(d) Doctors wanted to determine whether dietary supplements could help people infected
with HIV.

They randomly put volunteers with HIV into two groups:


• a treatment group, who received HIV medication and additional vitamin and mineral
supplements
• a control group, who received HIV medication but no additional supplements.

The details of the two groups are outlined in Table 4.1.

Table 4.1

treatment group control group


total number of volunteers 18 22
average age / years 45.6 46.6
average mass / kg 82.3 82.5

The dietary supplements were given to the treatment group twice a day for three months. The
nutrients in the supplements included:
• vitamin C
• vitamin D
• calcium
• iron
• other minerals and vitamins.

(i) Explain why vitamin C and iron are important in the human diet.

...........................................................................................................................................

...........................................................................................................................................

...........................................................................................................................................

...........................................................................................................................................

...........................................................................................................................................

...........................................................................................................................................

...........................................................................................................................................

...........................................................................................................................................

..................................................................................................................................... [4]

© UCLES 2021 0610/43/O/N/21


15

Table 4.2 shows some of the results from the study.

Table 4.2

treatment group control group


after three after three
at the start at the start
months months
average number of
357 422 461 461
lymphocytes / cells per μg of blood
average number of copies of HIV
4291 897 2648 5935
RNA per cm3 of blood

(ii) Use the data for the treatment group, shown in Table 4.2, to calculate the percentage
decrease in the average number of copies of HIV RNA per cm3 of blood.

Space for working.

.............................................................%
[2]

(iii) Evaluate the effect of the dietary supplements on the lymphocytes.

Use the information in Table 4.2 in your answer.

...........................................................................................................................................

...........................................................................................................................................

...........................................................................................................................................

...........................................................................................................................................

..................................................................................................................................... [2]

[Total: 16]

© UCLES 2021 0610/43/O/N/21 [Turn over


16

5 Fig. 5.1 shows people fishing on a large scale and a small scale.

large-scale fishing small-scale fishing

Fig. 5.1

All types of fishing can have a negative impact on fish stocks.

(a) Discuss how governments can regulate fishing to maintain fish stocks.

Use the word sustainable in your answer.

...................................................................................................................................................

...................................................................................................................................................

...................................................................................................................................................

...................................................................................................................................................

...................................................................................................................................................

...................................................................................................................................................

...................................................................................................................................................

...................................................................................................................................................

...................................................................................................................................................

...................................................................................................................................................

...................................................................................................................................................

...................................................................................................................................................

............................................................................................................................................. [6]

© UCLES 2021 0610/43/O/N/21


17

(b) Fig. 5.2 shows the location of a chemical factory near a river.

site 1

direction of
water flow

site 2

Fig. 5.2

Fig. 5.3 shows the sex ratio of the fish, Catostomus commersonii, in the river at site 1 and
site 2. Intersex fish have both female and male reproductive organs.

Key:
female
site 1
intersex
male

site 2

0 10 20 30 40 50 60 70 80 90 100
percentage of fish

Fig. 5.3

State and explain what type of chemical the factory could be releasing into the river that
would cause the effects shown in Fig. 5.3.

type of chemical ........................................................................................................................

explanation ...............................................................................................................................

...................................................................................................................................................

...................................................................................................................................................
[2]

(c) State how sex is inherited in humans.

............................................................................................................................................. [1]

[Total: 9]
© UCLES 2021 0610/43/O/N/21 [Turn over
18

6 The heart pumps blood around the body.

(a) Explain why the heart is an organ.

...................................................................................................................................................

...................................................................................................................................................

............................................................................................................................................. [1]

(b) Complete the sentences:

The ............................................... system includes the heart and blood vessels. Deoxygenated

blood from the body is transported to the heart in the ............................................... .

During a heart beat the ventricles contract. The right ventricle pumps deoxygenated blood to

the lungs. The right ventricle has a ............................................... muscular wall than the left

ventricle.

Gas exchange in the lungs occurs by ............................................... . Oxygenated blood

travels back to the heart where it enters the ............................................... of the heart.

The two sides of the heart are separated by the ............................................... . This

structure prevents the mixing of oxygenated and deoxygenated blood. Oxygenated blood is

then delivered to the rest of the body. Blood is supplied to the muscle of the heart in the

............................................... .
[7]

(c) Many people monitor their heart rate by counting their pulse.

State one other method of monitoring heart rate.

............................................................................................................................................. [1]

[Total: 9]

© UCLES 2021 0610/43/O/N/21


Cambridge IGCSE™
* 9 1 0 9 2 7 0 9 4 9 *

BIOLOGY 0610/43
Paper 4 Theory (Extended) October/November 2022

1 hour 15 minutes

You must answer on the question paper.

No additional materials are needed.

INSTRUCTIONS
● Answer all questions.
● Use a black or dark blue pen. You may use an HB pencil for any diagrams or graphs.
● Write your name, centre number and candidate number in the boxes at the top of the page.
● Write your answer to each question in the space provided.
● Do not use an erasable pen or correction fluid.
● Do not write on any bar codes.
● You may use a calculator.
● You should show all your working and use appropriate units.

INFORMATION
● The total mark for this paper is 80.
● The number of marks for each question or part question is shown in brackets [ ].

This document has 24 pages. Any blank pages are indicated.

DC (PQ/CGW) 302493/6
© UCLES 2022 [Turn over
2

1 (a) Fig. 1.1 is a diagram showing parts of two organ systems that are active when a person
sneezes.

O
L

view of part of the gas exchange system view of part of the nervous system

not to scale

Fig. 1.1

(i) Sneezing is an automatic action that occurs in response to a stimulus in the nose.

State the name of this type of automatic action.

..................................................................................................................................... [1]

© UCLES 2022 0610/43/O/N/22


3

(ii) Table 1.1 shows the names and functions of some of the parts of the human body that
are involved when a person sneezes, and the letters in Fig. 1.1 that identify these parts.

Complete Table 1.1.

Table 1.1

function name of structure letter in Fig. 1.1

cell that transmits an impulse from the


receptor to the central nervous system

diaphragm

contains cilia to move mucus out of the airway

[5]

© UCLES 2022 0610/43/O/N/22 [Turn over


4

(b) Fig. 1.2 is a diagram of a cell from the human nervous system.

Fig. 1.2

(i) On Fig. 1.2, label two visible cell structures. [2]

(ii) Draw an X on the cell in Fig. 1.2 to show where a receptor molecule for a neurotransmitter
would be found. [1]

(iii) Explain how the cell in Fig. 1.2 is adapted for transmitting impulses.

...........................................................................................................................................

...........................................................................................................................................

...........................................................................................................................................

...........................................................................................................................................

..................................................................................................................................... [2]

(c) Describe how nervous communication differs from hormonal communication.

...................................................................................................................................................

...................................................................................................................................................

...................................................................................................................................................

...................................................................................................................................................

...................................................................................................................................................

...................................................................................................................................................

............................................................................................................................................. [3]

[Total: 14]

© UCLES 2022 0610/43/O/N/22


5

2 (a) Digestive enzymes are secreted into the alimentary canal.

(i) As well as enzymes, other substances that are important for digestion are also secreted
into the alimentary canal.

State the names of two of these other substances.

1 ........................................................................................................................................

2 ........................................................................................................................................
[2]

(ii) Table 2.1 summarises some facts about the enzymes that are secreted into the alimentary
canal.

Complete Table 2.1.

Table 2.1

products of
organ where the organ when the
name of the enzyme digestion involving this
enzyme is secreted enzyme acts
enzyme

salivary glands

pepsin

pancreas fatty acids and glycerol

epithelial lining of the


small intestine
small intestine
[4]

© UCLES 2022 0610/43/O/N/22 [Turn over


6

(b) The activity of the enzyme pepsin was measured at different temperatures.

The results are shown in Fig. 2.1.

3000

2500

2000

enzyme activity
1500
/ U per mg protein

1000

500

0
10 20 30 40 50 60 70
temperature / °C

Fig. 2.1

© UCLES 2022 0610/43/O/N/22


7

Describe and explain the effect of temperature on the activity of pepsin.

Use the information in Fig. 2.1 to support your answer.

...................................................................................................................................................

...................................................................................................................................................

...................................................................................................................................................

...................................................................................................................................................

...................................................................................................................................................

...................................................................................................................................................

...................................................................................................................................................

...................................................................................................................................................

...................................................................................................................................................

...................................................................................................................................................

...................................................................................................................................................

...................................................................................................................................................

............................................................................................................................................. [6]

(c) Lactase is another digestive enzyme, which is produced by human babies.

(i) Explain why lactase is important for the nutrition of young babies.

...........................................................................................................................................

...........................................................................................................................................

...........................................................................................................................................

...........................................................................................................................................

..................................................................................................................................... [2]

(ii) Most people stop producing lactase when they become adults. If they drink milk, this
may give them diarrhoea.

Describe the symptoms of diarrhoea.

...........................................................................................................................................

...........................................................................................................................................

..................................................................................................................................... [1]

© UCLES 2022 0610/43/O/N/22 [Turn over


8

(iii) Cholera can also cause diarrhoea.

Explain how the cholera bacterium causes diarrhoea.

...........................................................................................................................................

...........................................................................................................................................

...........................................................................................................................................

...........................................................................................................................................

...........................................................................................................................................

...........................................................................................................................................

..................................................................................................................................... [3]

[Total: 18]

© UCLES 2022 0610/43/O/N/22


10

3 Fig. 3.1 shows the estimated growth of the global human population between the 1860s and
2010s.

It also shows the estimated number of people that were affected by famines between these dates.

22

20

18

16 8

14 7
estimated
number of 12 6
people affected
by famines 10 5
/ million estimated
global human
8 4 population
/ billion
6 3

4 2

2 1

0 0
18 0s
18 0s
18 0s
19 0s
19 0s
19 0s
19 0s
19 0s
19 0s
19 0s
19 0s
19 0s
19 0s
20 0s
20 0s
s
10
6
7
8
9
0
1
2
3
4
5
6
7
8
9
0
18

time / decade

1 billion = 1 000 000 000

Key:
estimated number of people affected by famines
estimated global human population

Fig. 3.1

(a) (i) State the final size of the estimated global human population shown in the 2010s in
Fig. 3.1.

..................................................................................................................................... [1]

© UCLES 2022 0610/43/O/N/22


11

(ii) Using the information in Fig. 3.1, calculate the percentage of the estimated global human
population that was affected by famine during the 1860s.

Give your answer to one significant figure.

............................................................ %
[2]

(b) An increase in the size of the human population can cause famine.

(i) Describe other factors that could have caused famines since the 1970s.

...........................................................................................................................................

...........................................................................................................................................

...........................................................................................................................................

...........................................................................................................................................

...........................................................................................................................................

...........................................................................................................................................

..................................................................................................................................... [3]

(ii) Chemical fertilisers have helped farmers to increase food production.

Discuss the negative impact on the natural environment of using chemicals, other than
fertilisers, in modern farming methods.

...........................................................................................................................................

...........................................................................................................................................

...........................................................................................................................................

...........................................................................................................................................

...........................................................................................................................................

...........................................................................................................................................

...........................................................................................................................................

...........................................................................................................................................

..................................................................................................................................... [4]

© UCLES 2022 0610/43/O/N/22 [Turn over


12

(iii) Crops are often grown as monocultures.

State what is meant by a monoculture.

...........................................................................................................................................

...........................................................................................................................................

..................................................................................................................................... [1]

(iv) Scientists are developing crop plants that can produce antigens so that people gain
immunity to different diseases such as hepatitis B.

State the process which can be used to develop crop plants that can produce proteins
from other species.

..................................................................................................................................... [1]

[Total: 12]

© UCLES 2022 0610/43/O/N/22


14

4 (a) Fig. 4.1 shows a dandelion plant, Taraxacum officinale, in a field. The flower stalk is called a
scape.

scape

Fig. 4.1

(i) The scape of a dandelion responds to sunlight by growing upwards.

State the name of this growth response.

..................................................................................................................................... [1]

(ii) The scapes of dandelions keep the plant upright without the need for structures such as
bones.

Explain how cells in plant scapes and stems keep plants upright.

...........................................................................................................................................

...........................................................................................................................................

...........................................................................................................................................

...........................................................................................................................................

..................................................................................................................................... [2]

© UCLES 2022 0610/43/O/N/22


15

(b) Fig. 4.2 is part of a cross-section through the scape of a dandelion.

Fig. 4.2

Draw a line and add a label on Fig. 4.2 to identify one area of xylem tissue. [1]

(c) Describe the function of phloem tissue.

...................................................................................................................................................

...................................................................................................................................................

...................................................................................................................................................

...................................................................................................................................................

...................................................................................................................................................

...................................................................................................................................................

............................................................................................................................................. [3]

© UCLES 2022 0610/43/O/N/22 [Turn over


16

(d) A dandelion scape was cut into long strips for an osmosis investigation.

Immediately after the scape was cut, the pieces of scape bent outwards, as shown in Fig. 4.3.

scape cut
into long strips

inner part of scape


that can absorb or lose water

outer part of scape that


cannot absorb or lose water

Fig. 4.3

Key:
A B inner sur
outer sur

C D

not to sca
© UCLES 2022 0610/43/O/N/22
that can absorb or lose water

17
outer part of scape that
cannot absorb
Strips or lose water
of dandelion scape were placed in four dishes, A, B, C and D, for 30 minutes.

Each dish contained a different concentration of salt solution.

Fig. 4.4 shows the appearance of the four strips of scape after 30 minutes.

Key:
A B inner surface of scape
outer surface of scape

C D

not to scale
Fig. 4.4

Using the information in Fig. 4.3 and Fig. 4.4, deduce and explain which strip of scape was in
the most concentrated salt solution.

strip of scape .............................................................................................................................

explanation ................................................................................................................................

...................................................................................................................................................

...................................................................................................................................................

...................................................................................................................................................

...................................................................................................................................................
[3]

[Total: 10]

© UCLES 2022 0610/43/O/N/22 [Turn over


18

5 (a) Researchers investigated water pollution in the centre of a lake for 11 days.

Fig. 5.1 shows researchers collecting samples of aquatic organisms from the lake with a net
which was placed at a specified water depth.

Fig. 5.1

Each day, the researchers collected 5 dm3 of water from the lake. Fish and other large
organisms were released back into the lake. The researchers extracted the chlorophyll from
all the microorganisms that were collected in the sample of lake water.

They determined the concentration of chlorophyll by measuring the mass of chlorophyll


extracted from each dm3 of lake water.

Fig. 5.2 shows a researcher lowering a Secchi disc on a long string into a lake from a boat.
A Secchi disc is a plastic disc which is divided into black and white sections. It is used to
measure the transparency of water in lakes, rivers and oceans.

The researcher recorded the maximum water depth at which the Secchi disc was still visible.

Secchi disc

Fig. 5.2

© UCLES 2022 0610/43/O/N/22


19

The results of the investigation are shown in Fig. 5.3.

6 1.0

5 1.5

4 2.0
maximum water
concentration depth at which
of chlorophyll 3 2.5
the Secchi disc
/ mg per dm3 is visible / m
2 3.0

1 3.5

0 4.0
1 2 3 4 5 6 7 8 9 10 11
day
Key:
concentration of chlorophyll
maximum water depth at which the Secchi disc is visible

Fig. 5.3

(i) State the type of microorganism that contains chlorophyll.

..................................................................................................................................... [1]

(ii) The researchers collected 5 dm3 of water from the lake each day. Using this information
and the information in Fig. 5.3, calculate the total mass of chlorophyll in the sample on
day 1.

Include the appropriate units with your answer.

................................................................
[2]

(iii) Suggest why the researchers determined the concentration of chlorophyll rather than
counting the number of microorganisms in the sample.

...........................................................................................................................................

...........................................................................................................................................

..................................................................................................................................... [1]

(iv) Using the information in Fig. 5.3, identify the day when the water was the most
transparent.

..................................................................................................................................... [1]

© UCLES 2022 0610/43/O/N/22 [Turn over


20

(v) Explain why the transparency of the water is important for organisms that contain
chlorophyll.

...........................................................................................................................................

...........................................................................................................................................

...........................................................................................................................................

...........................................................................................................................................

..................................................................................................................................... [2]

(b) (i) Heavy rainfall in nearby fields caused ions from fertilisers to wash into the lake on day 1
of the investigation.

State which ions can cause an increase in the number of organisms that contain
chlorophyll in the lake.

..................................................................................................................................... [1]

(ii) Many of the organisms that contain chlorophyll died on day 2 of the investigation.

Explain the consequences of the death of these organisms to the lake ecosystem.

...........................................................................................................................................

...........................................................................................................................................

...........................................................................................................................................

...........................................................................................................................................

...........................................................................................................................................

...........................................................................................................................................

..................................................................................................................................... [3]

© UCLES 2022 0610/43/O/N/22


21

(c) The researchers monitored the water in the lake for pollution.

Outline how sewage should be treated so that the water it contains is safe to put into the lake.

...................................................................................................................................................

...................................................................................................................................................

...................................................................................................................................................

...................................................................................................................................................

...................................................................................................................................................

...................................................................................................................................................

...................................................................................................................................................

...................................................................................................................................................

...................................................................................................................................................

...................................................................................................................................................

............................................................................................................................................. [5]

[Total: 16]

© UCLES 2022 0610/43/O/N/22 [Turn over


22

6 Plants can be classified according to the position and shape of the structures in their flowers.

(a) Fig. 6.1 shows sections of flowers from six different plant species.

A B

C D

E F

Fig. 6.1

© UCLES 2022 0610/43/O/N/22


23

(i) Use the key to identify each species. Write the letter of each species (A to F) in the
correct box in the key.

key

1(a) two petals visible go to 2

(b) three petals visible go to 4

2(a) one stigma per flower go to 3

(b) more than one stigma per flower Pyrus communis

3(a) stigma higher than anther Prunus domestica

(b) stigma at same level or lower than anther Prunus salicina

4(a) two ovules visible go to 5

(b) more than two ovules visible Punica granatum

5(a) anther smaller than carpel Prunus amygdalus

(b) anther larger than carpel Olea europaea


[4]

(ii) The large petals of the flowers shown in Fig. 6.1 indicate that they are pollinated by
insects.

List four other structural adaptations of insect-pollinated flowers.

1 ........................................................................................................................................

2 ........................................................................................................................................

3 ........................................................................................................................................

4 ........................................................................................................................................
[4]

(b) Flowering plants are distinguished from ferns because they all produce flowers.

State one other morphological feature that can be used to distinguish flowering plants from
ferns.

............................................................................................................................................. [1]

(c) The fruit of a pear tree, Pyrus communis, is often used to make juice.

State what protein biotechnologists will use to increase the volume of juice produced from
pear fruit.

............................................................................................................................................. [1]

[Total: 10]

© UCLES 2022 0610/43/O/N/22

You might also like